You are on page 1of 51

1. With reference to archaeological mounds, which of the following statements is/are correct?

1. The archaeological remains are better preserved particularly in the middle Gangetic plains.
2. Large scale remains of the burnt brick phase are found in the riverine and delta areas.
Select the correct answer using the code given below.
(a) 1 only
(b) 2 only
(c) Both 1 and 2
(d) Neither 1 nor 2

Answer: (b)
Explanation: Archaeological remains are generally in a better state of preservation in dry arid
climate. Ex: West U.P, Rajasthan and Northwest India. In the moist and humid climate of middle
Gangetic plains and the delta regions, even the iron implements suffer corrosion. Mud
structures are difficult to detect. Only during the phase of burnt brick structures, large scale
remains are found in these areas. Statement 1 is incorrect. Statement 2 is correct.
Reference: Page: 11; Ancient India, NCERT-XI

2. With reference to the sources of history, which of the following statements is/are incorrect?
1. Large number of coin moulds, made of clay, belong to the Gupta period.
2. The first coins to clearly mention the details of the king and the year, belong to the Mauryas.
Select the answer using the code given below.
(a) 1 only
(b) 2 only
(c) Both 1 and 2
(d) Neither 1 nor 2

Answer: (c)
Explanation: Large number of burnt clay moulds have been found. They belong to the Kushan
period (0-300 AD). They are absent during the Gupta period.
The early coins had symbols. The coins of the Indo-Greeks were the first to contain the details of
the king and the year. This would help reconstruct the history of ruling dynasties.
Both Statements are incorrect.
Reference: Page: 12, 13; Ancient India, NCERT-XI

3. With reference to the sources of history, which of the following statements is/are correct?
1. The largest number of gold coins were issued by the Guptas.
2. The post-Gupta period has the largest number of coins.
Select the correct answer using the code given below.
(a) 1 only
(b) 2 only
(c) Both 1 and 2
(d) Neither 1 nor 2

Answer: (a)
Explanation: The largest number of coins had been found in the post Maurya age. This was an
age when trade and commerce flourished. The post-Gupta age is known for decline in trade and
commerce, indicated by the presence of very few coins. Statement 2 is incorrect.
The largest number of gold coins had been found in the strata corresponding to the Gupta
period. Statement 1 is correct.
Reference: Page: 13; Ancient India, NCERT-XI
4. With reference to inscriptions as the sources of history, which of the following statements
is/are incorrect?

1. The earliest inscriptions found, belong to the Harappan period.


2. The oldest inscription deciphered so far, belong to the Ashokan period.
Select the answer using the code given below.
(a) 1 only
(b) 2 only
(c) Both 1 and 2
(d) Neither 1 nor 2

Answer: (d)
Explanation: Harappan seals are the earliest inscriptions found in India. They belong to the
period of about 2500 BC. They have not been deciphered. They have a pictorial script. They are
written in boustrophedon script (right to left and left to right alternatingly).
Ashokan inscriptions belonging to the third century, are the earliest to have been deciphered.
They were first deciphered by James Princep, an East India Company employee.
Both Statements are correct.
Reference: Page: 14; Ancient India, NCERT-XI

5. With reference to inscriptions as the sources of history, which of the following statements
is/are correct?
1. Brahmi script is written from left to right.
2. Kharosthi script is written from left to right.
Select the correct answer using the code given below.
(a) 1 only
(b) 2 only
(c) Both 1 and 2
(d) Neither 1 nor 2

Answer: (a)
Explanation: Brahmi script is written from left to right. It continued to be the main script used
till the Gupta period.
Kharosthi script is written from right to left. They are found in Ashokan inscriptions found in
the north west parts of the subcontinent.
Statement 1 is correct. Statement 2 is incorrect.
Reference: Page: 14; Ancient India, NCERT-XI

6. With reference to the literary sources of history, which of the following statements is/are
correct?
1. Srautasutras describe the various kinds of measurements for the construction of sacrificial
altars.
2. Sulvasutras mention and describe the royal and public ceremonies.
Select the correct answer using the code given below.
(a) 1 only
(b) 2 only
(c) Both 1 and 2
(d) Neither 1 nor 2

Answer: (d)
Explanation: Ritual literature is an important part of post-Vedic literature. Srauta sutras
mention and describe the royal and public ceremonies. Grihya sutras mention and describe the
domestic ceremonies, to be done in one's home. Sulva sutras describe the various kinds of
measurements for the construction of sacrificial altars. They have notations related to geometry
and mathemetics.
Both Statements are incorrect.
Reference: Page: 16; Ancient India, NCERT-XI

7. With reference to the literary sources of history, which of the following statements is/are
incorrect?
1. Grihyasutras mention and describe domestic ceremonies.
2. Sulvasutra is the first source which has geometrical notations.
Select the answer using the code given below.
(a) 1 only
(b) 2 only
(c) Both 1 and 2
(d) Neither 1 nor 2

Answer: (d)
Explanation: Ritual literature is an important part of post-Vedic literature. Srauta sutras
mention and describe the royal and public ceremonies. Grihya sutras mention and describe the
domestic ceremonies, to be done in one's home. Sulva sutras describe the various kinds of
measurements for the construction of sacrificial altars. They have notations related to geometry
and mathemetics.
Both Statements are correct.
Reference: Page: 16; Ancient India, NCERT-XI

8. With reference to the literary sources of history, which of the following statements is/are
correct?
1. Sangam literature does not constitute religious literature.
2. Dharmasutras and Smritis are considered parts of religious literature.
Select the correct answer using the code given below.
(a) 1 only
(b) 2 only
(c) Both 1 and 2
(d) Neither 1 nor 2

Answer: (a)
Explanation: Dharmasutras, Smritis and Dharmashastras (commentaries) belong to a class of
secular literature. They are essentially lawbooks.
Sangam literature are secular in nature. They have much information about the social and
economic life of the Sangam Tamils. Pastoral wars, military exploits form the PURAM part
whereas love, separation and other aspects form the AKAM part.
Statement 2 is incorrect. Statement 1 is correct.
Reference: Page: 17, 18; Ancient India, NCERT-XI

9. With reference to the literary sources of history, which of the following statements is/are
correct?
1. The biography of Ramapala named Ramacharita, was written by Sandhyakara Nandi.
2. Mushika Vamsha, written by Atula in eleventh century, is about the Chalukyas of Kalyan.
3. Rajatarangini was written by Bilhana about the all the dynasties of Kashmir.
Select the correct answer using the code given below.
(a) 2 and 3
(b) 3 only
(c) 1 only
(d) 1 and 3

Answer: (c)
Explanation: Ramacharita, written by Sandhyakara Nandi in twelfth century, narrates the
biography of the Pala king Ramapala. Statement 1 is correct.
Mushika Vamsa, written by Atula in eleventh century, is an account of the Mushikas who ruled
the northern Kerala. Statement 2 is incorrect.
Rajatarangini, written by Kalhana in twelfth century, gives an account of the dynasties of
Kashmir.
Vikramanakadevacharita, written by Bilhana, recounts the biography of Chalukya king
Vikramaditya VI of Kalyan. Statement 3 is incorrect.
Reference: Page: 20; Ancient India, NCERT-XI

10. With reference to the periods of history, which of the following statements is/are correct?
1. Vikrama Samvat began in 78 AD.
2. Shaka Samvat began in 57 BC.
3. Gupta era began in 319 AD.
Select the correct answer using the code given below.
(a) 2 and 3 only
(b) 3 only
(c) 1 and 2 only
(d) 1, 2 and 3

Answer: (b)
Explanation: The Shakas entered India and established as many as 5 branches. The king of
Ujjain, Vikramaditya, defeated them and on the event of his victory, an era called Vikrama
Samvat was started in 57 BC. Statement 1 is incorrect.
The Kushana king Kanishka started the Shaka era in 78 AD. Statement 2 is incorrect.
Chandragupta I started the the Gupta dynasty in 319-20 AD. This came to be known as the
Gupta era. Statement 3 is correct.
Reference: Page: 19, 143, 144, 181; Ancient India, NCERT-XI

11. With reference to the phases in Paleolithic age, which of the following statements
is/are correct?
1. Hand axes and cleavers belong to the Lower Paleolithic age.
2. The Upper Paleolithic age industries are based on flakes.
Select the correct answer using the code given below.
(a) 1 only
(b) 2 only
(c) Both 1 and 2
(d) Neither 1 nor 2

Answer: (a)
Explanation: Lower Paleolithic age (500000 BC - 50000 BC) covers the greater part of the Ice
age. Hand-axes, cleavers and choppers are characteristic of this period. In India, sites pertaining
to this period are found in Sohan valley of Punjab, Kashmir, Thar desert, Belan valley of
Mirzapur (U.P), Narmada valley. Statement 1 is correct.
Middle Paleolithic age (50000 BC - 40000 BC) sites are found in Narmada valley and south of
Tungabhadra. The industries are based on flakes. The main tools are blades, points, borers,
scrapers and blade like tools. Statement 2 is incorrect.
Upper Paleolithic age (40000 BC - 10000 BC) phase was less humid. It marks the appearance of
flint industries and Homo sapiens (modern man). Blades and burins are found. The sites are
found in A.P, Karnataka, Maharashtra, Madhya Pradesh and southern parts of U.P and Bihar.
Reference: Page: 34, 35; Ancient India, NCERT-XI

12. With reference to the stone ages, which of the following statements is/are correct?
1. Paleolithic age sites are generally found in the alluvial plains.
2. The characteristic tools found in Neolithic Age are microliths.
Select the correct answer using the code given below.
(a) 1 only
(b) 2 only
(c) Both 1 and 2
(d) Neither 1 nor 2

Answer: (d)
Explanation: Paleolithic sites are generally found in the hilly slopes and river valleys. They are
absent in the alluvial plains, which are inhabited at a later time.
Microliths are characteristic of the Mesolithic Age. The Mesolithic sites are found in Rajasthan,
central and Eastern India and south of river Krishna. Adamgarh, Bagor and Bhimbetka are
important sites.
Neolithic Age is generally associated with agriculture. Polished stone implements are
considered characteristic of this period.
Both Statements are incorrect.
Reference: Page: 35, 37; Ancient India, NCERT-XI

13. With reference to the Neolithic Age, which of the following statements is/are correct?

1. Mehrgarh is the earliest site of this age found in the subcontinent.


2. Neolithic settlements of south are generally older than the ones in north.
Select the correct answer using the code given below.
(a) 1 only
(b) 2 only
(c) Both 1 and 2
(d) Neither 1 nor 2

Answer: (a)
Explanation: Neolithic Age is generally associated with agriculture. Polished stone implements
are considered characteristic of this period. Mehrgarh is the earliest site of this age found in the
subcontinent. It is attributed to 7000 BC and is found in Baluchistan. Statement 1 is correct.
Neolithic settlements in south India are not older than 2500 BC. A few sites in Vindhyas are as
old as 5000 BC. The older sites are found in north and north-western part. Statement 2 is
incorrect.
Reference: Page: 36, 37; Ancient India, NCERT-XI

14. With reference to the Neolithic age in Kashmir, which of the following statements is/are
incorrect?
1. It is characterised by the presence of dwelling pits.
2. It is marked by an absence of microliths.
Select the answer using the code given below.
(a) 1 only
(b) 2 only
(c) Both 1 and 2
(d) Neither 1 nor 2

Answer: (d)
Explanation: Kashmiri neolithic culture is marked by dwelling pits, ceramics, stone and bone
tools, absence of microliths. Burzahom and Gufkral are important sites.
Both Statements are correct.
Reference: Page: 37; Ancient India, NCERT-XI

15. With reference to the Neolithic sites in India, which of the following statements
is/are correct?
1. Burzahom is a site associated with bone implements.
2. Chirand is associated with the burial of dogs along with human beings.
Select the correct answer using the code given below.
(a) 1 only
(b) 2 only
(c) Both 1 and 2
(d) Neither 1 nor 2

Answer: (a)
Explanation: Kashmiri neolithic culture is marked by dwelling pits, ceramics, stone and bone
tools, absence of microliths. Burzahom and Gufkral are important sites. Burzahom is generally
associated with the burial of dogs with human beings. No other neolithic site in India has this
feature. Statement 1 is correct.
Chirand, a neolithic site in Bihar. It is known for its bone implements industry. Statement 2 is
incorrect.
Reference: Page: 37, 38; Ancient India, NCERT-XI

16. With reference to the stone ages, which of the following statements is/are correct?
1. Ash mounds are particularly associated with Mesolithic age.
2. Neolithic settlements are not found in the north-east at all.
Select the correct answer using the code given below.
(a) 1 only
(b) 2 only
(c) Both 1 and 2
(d) Neither 1 nor 2

Answer: (d)
Explanation: Ash mounds are associated with Neolithic age. The cattle herders used to set up
seasonal camps surrounded by cowpens. Dung used to accumulate in these enclosures. They are
put to fire while clearing the ground for the next season. Piklihal is an important site.
Neolithic settlements are found in 3 distinct areas in India - northwest, south of Godavari and
northeast. In the northeast, they are found in Assam, Garo hills.
Both Statements are incorrect.
Reference: Page: 38, 40; Ancient India, NCERT-XI
17. With reference to the Neolithic age, which of the following statements is/are incorrect?
1. Pottery appeared for the first time in human history during this period.
2. Wheat and cotton were produced in the subcontinent during this period.
Select the answer using the code given below.
(a) 1 only
(b) 2 only
(c) Both 1 and 2
(d) Neither 1 nor 2

Answer: (d)
Explanation: Neolithic age is generally associated with agriculture. Polished stone implements
are considered characteristic of this period. Mehrgarh is the earliest site of this age found in the
subcontinent. It is attributed to 7000 BC and is found in Baluchistan. Wheat, cotton were
produced by these people. Pots were required to store the food grains. Hence pottery appeared
first in this period.
Both Statements are correct.
Reference: Page: 40; Ancient India, NCERT-XI

18. With reference to the early Vedic Aryans, which of the following statements is/are
correct?
1. The economy was primarily agrarian.
2. Horse played a significant role in their life.
Select the correct answer using the code given below.
(a) 1 only
(b) 2 only
(c) Both 1 and 2
(d) Neither 1 nor 2
Answer: (b)
Explanation: Rig Vedic society was primarily pastoral in nature. Agriculture was a secondary
occupation. Their lifestyle was not sedentary. Statement 1 is incorrect.
Domesticated horse was found in Black Sea and Ural mountains around 6000 BC. The Aryans
originally came from the steppes of southern Russia, through central Asia and Iran to the
northwest of India. The presence of horse aided them to move swiftly. Statement 2 is correct.
Reference: Page: 70; Ancient India, NCERT-XI

19. With reference to the early Vedic Aryans, which of the following statements is/are
correct?
1. Early Aryans appeared in the subcontinent around 1500 BC.
2. Early Aryans settled in the fertile alluvial plains of Indus and Ganges.
Select the correct answer using the code given below.
(a) 1 only
(b) 2 only
(c) Both 1 and 2
(d) Neither 1 nor 2

Answer: (a)
Explanation: Early Aryans were found in the regions of East Afghanistan, North-West Frontier
Province (Punjab of Pakistan), Punjab. They migrated to the Gangetic plains later. They
appeared in the subcontinent around 1500 BC.
Statement 1 is correct. Statement 2 is incorrect.
Reference: Page: 70, 71; Ancient India, NCERT-XI

20. With reference to Rig Veda, which of the following statements is/are incorrect?
1. Rig Veda is the earliest text in all of the Indo-European languages.
2. Rig Vedic terms occur in the Iranian text, Avesta.
Select the answer using the code given below.
(a) 1 only
(b) 2 only
(c) Both 1 and 2
(d) Neither 1 nor 2

Answer: (d)
Explanation: Rig Veda consists of 10 mandalas or books. It is the earliest book in the Indo-
European languages. Books II to VII are the earliest. Books I and X are later additions. Rig Veda
has many terms common with Avesta, the earliest text in Iranian language.
Both statements are correct.
Reference: Page: 70; Ancient India, NCERT-XI

21. With reference to the Rig Vedic society, which of the following statements is/are
correct?
1. Land was not an established form of private property.
2. A system of taxation was absent.
Select the correct answer using the code given below.
(a) 1 only
(b) 2 only
(c) Both 1 and 2
(d) Neither 1 nor 2h

Answer: (c)
Explanation: Rig Vedic society was primarily pastoral. Agriculture was a secondary occupation.
Lands were used for grazing, cultivation and settlement but it was not an established form of
private property.
There are no references to any officer collecting taxes. The spoils of war were distributed in
some Vedic assemblies. Voluntary offerings called Bali were received from the people.
Both Statements are correct.
Reference: Page: 72, 74; Ancient India, NCERT-XI

22. With reference to the Rig Vedic society, which of the following statements is/are incorrect?
1. Women were prohibited to attend the public assemblies.
2. The wars were fought between different tribes among Aryans mainly for the control of the
land.
Select the answer using the code given below.
(a) 1 only
(b) 2 only
(c) Both 1 and 2
(d) Neither 1 nor 2

Answer: (c)
Explanation: Sabha, Samiti, Vidatha and Gana are the Vedic assemblies. During the early Vedic
period, women were allowed to attend Sabha and Vidatha. During the later Vedic period, entry
of women was restricted.
Most wars were mainly fought for the sake of cows. War was called, gavishthi, search for cows.
The king or rajan was the chief of the tribes. He was attached to the people and not the land.
Both statements are incorrect.
Reference: Page: 72, 73; Ancient India, NCERT-XI
23. With reference to the Rig Vedic period, which of the following statements is/are
correct? 1. Child marriage was highly prevalent.
2. Rig Vedic hymns discriminate against the birth of a girl child.
Select the correct answer using the code given below.
(a) 1 only
(b) 2 only
(c) Both 1 and 2
(d) Neither 1 nor 2

Answer: (b)
Explanation: Rig Vedic hymns show the desire for children and cattle. The birth of son was
desired repeatedly so that they could fight wars. The birth of a daughter was not desired upon.
Child marriage was not found. Marriageable age seems to be 16 or 17 during this period.
Statement 1 is incorrect. Statement 2 is correct.
Reference: Page: 74; Ancient India, NCERT-XI

24. With reference to the Rig Vedic period, which of the following statements is/are incorrect?
1. There was no territorial administration.
2. There was no standing army.
Select the answer using the code given below.
(a) 1 only
(b) 2 only
(c) Both 1 and 2
(d) Neither 1 nor 2

Answer: (d)
Explanation: Rig Vedic society was primarily pastoral. Agriculture was a secondary occupation.
Lands were used for grazing, cultivation and settlement but it was not an established form of
private property. The administration was tribal. People were migrating most of the time. So
there was no territorial administration.
There was no regular or standing army. In times of war, the chief mustered militia.
Both Statements are correct.
Reference: Page: 74; Ancient India, NCERT-XI

25. With reference to the early Vedic Aryans, which of the following statements is/are
correct?
1. Prayers and sacrifice was the chief mode of worship.
2. Prayers were accompanied by rituals and sacrificial formulae.
Select the correct answer using the code given below.
(a) 1 only
(b) 2 only
(c) Both 1 and 2
(d) Neither 1 nor 2

Answer: (a)
Explanation: There were no temples during this period. The main form of worship was
recitation of prayers and offering of sacrifices. They were not accompanied by rituals or
sacrificial formulae. They were the developments of the later Vedic age.
Statement 1 is correct. Statement 2 is incorrect.
Reference: Page: 76, 77; Ancient India, NCERT-XI
Art and Culture:
26. With reference to prehistoric art, which of the following statements is/are correct?
1. The earliest prehistoric paintings in India are found only from the Mesolithic period.
2. The richest paintings are found in the Vindhyan ranges.
Select the correct answer using the code given below.
(a) 1 only
(b) 2 only
(c) Both 1 and 2
(d) Neither 1 nor 2

Answer: (b)
Explanation: Prehistoric paintings are found during seven historical periods, in India. The
earliest of this belongs to the Upper Paleolithic period. They are continued in the Mesolithic and
Chalcolithic periods too. Statment 1 is incorrect.
The richest prehistoric paintings are found in Bhimbetka, Madhya Pradesh. It is located in the
Vindhyan ranges. It has about 800 rock shelters 500 of them have paintings. Statement 2 is
correct.
Reference: Page: 1, 3; An introduction to Indian art, NCERT-XI

27. With reference to prehistoric art, which of the following statements is/are correct?
1. Plants and animals are equally well exhibited in the prehistoric paintings.
2. Geometric patterns form an important category of prehistoric paintings.
Select the correct answer using the code given below.
(a) 1 only
(b) 2 only
(c) Both 1 and 2
(d) Neither 1 nor 2

Answer: (b)
Explanation: The three main categories of paintings: man, animal and geometric patterns. The
subjects depicted are bull, elephant, sambhar deer, gazelle, sheep, goat, horse, stylised humans,
trident but vegetal motifs are rarely depicted.
Statment 1 is incorrect. Statement 2 is correct.
Reference: Page: 2, 3; An introduction to Indian art, NCERT-XI

28. With reference to prehistoric art, which of the following statements is/are correct?
1. The paintings of upper Paleolithic period are mostly geometric patterns.
2. The paintings of upper Paleolithic period have only black and white colouration.
Select the correct answer using the code given below.
(a) 1 only
(b) 2 only
(c) Both 1 and 2
(d) Neither 1 nor 2

Answer: (a)
Explanation: The paintings belonging to the earliest upper Paleolithic period are mostly filled
with geometric patterns. They have stick like human figures. The dancers are represented in
green and the hunters in red.
Statment 1 is correct. Statement 2 is incorrect.
Reference: Page: 3, 4; An introduction to Indian art, NCERT-XI
29. With reference to prehistoric art, which of the following statements is/are correct?
1. The largest number of prehistoric paintings belong to the Upper Paleolithic period.
2. Hunting and community dance are two important themes of Mesolithic period.
Select the correct answer using the code given below.
(a) 1 only
(b) 2 only
(c) Both 1 and 2
(d) Neither 1 nor 2

Answer: (b)
Explanation: Prehistoric paintings are found during seven historical periods, in India. The
earliest of this belongs to the Upper Paleolithic period. They are continued in the Mesolithic and
Chalcolithic periods too. The largest number of prehistoric paintings belong to the Mesolithic
period.
Hunting scenes and community dances are important themes of Mesolithic period.
Statment 1 is incorrect. Statement 2 is correct.
Reference: Page: 4; An introduction to Indian art, NCERT-XI

30. With reference to prehistoric art, which of the following statements is/are correct?
1. Humans are depicted very naturally in the Mesolithic period.
2. Hand prints and fist prints are found in many shelters of Mesolithic period.
Select the correct answer using the code given below.
(a) 1 only
(b) 2 only
(c) Both 1 and 2
(d) Neither 1 nor 2

Answer: (b)
Explanation: Animals were painted in a naturalistic style but the humans were depicted only in
stylistic manner.
Hand prints, fist prints, dots made by finger prints are part of the paintings found in rock
shelters.
Statment 1 is incorrect. Statement 2 is correct.
Reference: Page: 4; An introduction to Indian art, NCERT-XI

31. With reference to Indus valley art, which of the following statements is/are correct?
1. There are abundant statues made of stone, representing Indus valley art.
2. The famous bust of the bearded man is made of steatite.
Select the correct answer using the code given below.
(a) 1 only
(b) 2 only
(c) Both 1 and 2
(d) Neither 1 nor 2

Answer: (b)
Explanation: Generally statues are not abundant in the Indus valley. But the art form is refined.
There are two famous statues of stone. One is a torso made of red sandstone and the other is the
bust of a bearded man made of steatite.
Statement 1 is incorrect. Statement 2 is correct.
Reference: Page: 9; An introduction to Indian art, NCERT-XI

32. With reference to Indus valley art, which of the following statements is/are correct?
1. The famous mother goddess statue is made of bronze.
2. Bronze statues are made using the lost wax technique.
Select the correct answer using the code given below.
(a) 1 only
(b) 2 only
(c) Both 1 and 2
(d) Neither 1 nor 2

Answer: (b)
Explanation: Bronze casting is well known to Harappans. They used 'lost wax' technique for
bronze statues. Wax figures were first covered with clay. As they were heated, the wax is
drained out using a hole. This hollow mould is then filled with molten metal. 'Dancing girl' and
buffalo are famous bronze statues.
The mother goddess statue is made of terracotta. Horned deity, toy carts, animals, gamesmen
are also found. Terracotta images are generally crude but abundant.
Statement 1 is incorrect. Statement 2 is correct.
Reference: Page: 10, 11; An introduction to Indian art, NCERT-XI

33. With reference to Indus valley art, which of the following statements is/are correct?
1. Seals are the chief source of Harappan script.
2. Bull, elephant and lion are found in Harappan seals.
Select the correct answer using the code given below.
(a) 1 only
(b) 2 only
(c) Both 1 and 2
(d) Neither 1 nor 2

Answer: (a)
Explanation: Harappan script is mainly found in the seals used in Indus valley. The seals are
engraved with the pictographic script. The script is yet to be deciphered.
Seals are the most widely found art forms of Harappan culture. Thousands of seals have been
found by archaeologists. They are made up of agate, chert, copper, faience, terracotta and even
gold and ivory. But the abundant number of seals is made up of steatite, a soft river stone. They
bear variety of motifs such as bull, elephant, tiger, goat, unicorn bull, rhinoceros, bison, goat,
buffalo, trees and human figurines. Lion is not found in Harappan seals.
Statement 1 is correct. Statement 2 is incorrect.
Reference: Page: 11; An introduction to Indian art, NCERT-XI

34. With reference to Indus valley art, which of the following statements is/are correct?
1. The Harappan seals were used for commerce.
2. The famous Pashupati seal consists of elephant, tiger, buffalo and rhinoceros.
Select the correct answer using the code given below.
(a) 1 only
(b) 2 only
(c) Both 1 and 2
(d) Neither 1 nor 2

Answer: (c)
Explanation: Seals are the most widely found art forms of Harappan culture. Thousands of seals
have been found by archaeologists. They are made up of agate, chert, copper, faience, terracotta
and even gold and ivory. But the abundant number of seals is made up of steatite, a soft river
stone. The purpose of these seals was mainly commercial. Sometimes they were used as amulets
too.
They bear variety of motifs such as bull, elephant, tiger, goat, unicorn bull, rhinoceros, bison,
goat, buffalo, trees and human figurines. The seal with a cross-legged human figure, usually
referred as Pashupati, has depictions of elephant, tiger, buffalo and rhinoceros, antelopes.
Both Statements are correct.
Reference: Page: 11, 12; An introduction to Indian art, NCERT-XI

35. With reference to Mauryan art, which of the following statements is/are correct?
1. Large number of statues of Yakshas and Yakshinis belong to the Mauryan period.
2. The worship of Yakshas and Yakshinis began only after the rise of Buddhism and Jainism.
Select the correct answer using the code given below.
(a) 1 only
(b) 2 only
(c) Both 1 and 2
(d) Neither 1 nor 2

Answer: (a)
Explanation: The worship of Yakshas and Yakshinis was prevalent during the Mauryan period
and it was assimilated by Buddhism and Jainism. But it was popular even before their advent.
Large number of statues of Yakshas and Yakshinis, made of polished stones were found
belonging to the Mauryan period. Patna, Vidisa, Mathura has many such works.
Statement 1 is correct. Statement 2 is incorrect.
Reference: Page: 19, 20; An introduction to Indian art, NCERT-XI

36. With reference to Quality council of India(QCI) , Consider the following statements.
1. Chairman of QCI is nominated by the Prime Minister.
2. QCI is an autonomous body under Department of Industrial Policy and Promotion, Ministry of
Commerce and Industry.
Which of the above statements is /are correct?
a. 1 only
b. 2 only
c. Both 1 and 2
d. Neither 1 nor 2

Answer: C
Explanation: Both the statements are correct.
The Quality Council of India (QCI) is a pioneering experiment of the Government of India in
setting up organizations in partnership with the Indian industry.
 The Mission of QCI is to lead nationwide quality movement in India by involving all
stakeholders for emphasis on adherence to quality standards in all spheres of activities
primarily for promoting and protecting interests of the nation and its citizens.
 It was set up jointly by the Government of India and the Indian Industry represented by the
three premier industry associations i.e. Associated Chambers of Commerce and Industry of
India (ASSOCHAM), Confederation of Indian Industry (CII) and Federation of Indian Chambers
of Commerce and Industry (FICCI).
 Chairman is nominated by Prime Minister.
 QCI is an autonomous body under Department of Industrial Policy and Promotion, Ministry
of Commerce and Industry. Secretary, Industrial Policy and Promotion is currently the
Chairman of QCI. Representatives from several government departments are Council members
of QCI. QCI does enjoy a cooperative relationship with all departments of Government.

Source: https://www.qcin.org/faq.php
37. Consider the following statements.
1. Kumbh Mela is held only once in 12 years at four different locations across Central and
Northern India.
2. The Kumbh Mela was inscribed on the list of “Intangible Cultural Heritage of Humanity” by
UNESCO in 2017.
3. Swachh Kumbh , is a cleanliness drive that was launched for people’s voluntary participation
in the Kumbh Mela 2019 , held at Prayagraj.
Which of the above statements is /are correct?
a. 1 and 2 only
b. 2 and 3 only
c. 1 only
d. 1 ,2 and 3

Answer: B
Explanation:Statement 1 is incorrect.The Kumbh Mela (the festival of the sacred pitcher) is
anchored in Hindu mythology. It is the largest public gathering and collective act of faith,
anywhere in the world. Kumbh Mela, in Hinduism, is a religious pilgrimage that is
celebrated four times over a course of 12 years. The geographical location of Kumbh Mela
spans over four locations in India and the Mela site keeps rotating between one of the four
pilgrimages on four sacred rivers as listed below:
 Haridwar on the Ganges in Uttarakhand
 Ujjain on the Shipra in Madhya Pradesh
 Nashik on the Godavari in Maharashtra
 Prayagraj at the confluence of the Ganges, the Yamuna, and the mythical Saraswati in
Uttar Pradesh (2019, Kumbh Mela)
Statement 2 and 3 are correct.Kumbh Mela is the intangible cultural heritage from India to be
listed in UNESCO’s list.A total of 13 Intangible cultural heritage (ICH) elements from India have
been inscribed till date on the UNESCO’s Representative List of the Intangible Cultural Heritage
of Humanity.
http://pib.nic.in/newsite/PrintRelease.aspx?relid=176981 - Representative List of the
Intangible Cultural Heritage of Humanity.
Source: https://kumbh.gov.in/en/key-notes
https://kumbh.gov.in/en/campaigns-swachh-kumbh
Refer Pulse Magazine 34 ( Jan 27 - Feb 2)

38. Consider the following statements regarding Indus Water Treaty.


1. According to treaty, the waters of eastern rivers shall be available for unrestricted use in
India.
2. India receives more percentage of water from the treaty than Pakistan.
Which of the above statements is /are correct?
a. 1 only
b. 2 only
c. Both 1 and 2
d. Neither 1 nor 2
Answer: A
Explanation: Statement 1 is correct.The Indus Water Treaty (IWT) is a water-distribution
treaty between India and Pakistan signed on September 19, 1960.
The Indus Waters Treaty (IWT) deals with river Indus and its five tributaries, which are
classified in 2 categories:
EASTERN RIVERS:
 Sutlej
 Beas
 Ravi
WESTERN RIVERS:
 Jhelum
 Chenab
 Indus
According to treaty, all the water of eastern rivers shall be available for unrestricted use in
India.
 India should let unrestricted flow of water from western rivers to Pakistan.
 It doesn’t mean that India can’t use western river’s water.
 The treaty says that India can use the water in western rivers in “non-consumptive” needs.
 Here non consumptive means we can use it for irrigation, storage and even for electricity
production.
Statement 2 is incorrect.The treaty allocates 80% of water from the six-river Indus water
system to Pakistan.

Why this question?


The Pakistan officials will visit the Chenab basin for inspection as per the Indus water
treaty,1960. As per the treaty, officials from both sides are mandated to inspect sites and works
on both sides of the Indus basin in a block of five years.

Source: https://mea.gov.in/bilateral-documents.htm?dtl/6439/Indus

https://en.m.wikipedia.org/wiki/Indus_Waters_Treaty

Refer Pulse Magazine 34 ( Jan 27 - Feb 2)

39. The National Tiger Conservation Authority has planned to reintroduce tigers in the
Satkosia Tiger reserve. Where is this reserve located?
a. Madhya Pradesh
b. Uttar Pradesh
c. Odisha
d. Chhattisgarh
Answer: C
Explanation:
 Tiger Reintroduction Two elephants are being brought from the Similipal Tiger Reserve to
strengthen Patrolling in the Satkosia Tiger Reserve, before approaching the NTCA for resuming
the ambitious tiger reintroduction programme in Satkosia. Programme The National Tiger
Conservation Authority has planned to reintroduce tigers in three tiger reserves, where their
number have dwindled to the minimum. Tiger restocking will happen at the Satkosia forest in
Odisha, the western part of Rajaji National Park in Uttarakhand and Buxa in West Bengal.
 For Satkosia reintroduction, the Madhya Pradesh government has given in-principle
approval to take out three pairs of breeding tigers from Panna.
 Earlier the Tiger reintroduction programme was carried out successfully in Sariska and
Panna Tiger Reserves, where their numbers have improved significantly.

Why this question?


Two elephants are being brought from the Similipal Tiger Reserve to strengthen Patrolling in
the Satkosia Tiger Reserve, before approaching the NTCA for resuming the ambitious tiger
reintroduction programme in Satkosia.

Source: https://www.google.com/amp/s/www.thehindu.com/news/national/other-
states/jumbos-to-patrol-odishas-satkosia-tiger-reserve/article26106411.ece/amp/

Refer Pulse Magazine 34 ( Jan 27 - Feb 2)

40. With reference to Program for International Student Assessment (PISA), Consider
the following statements.
1. PISA measures students’ performance in mathematics, reading, and science literacy.
2. PISA is coordinated by PRATHAM foundation, a non-governmental organization.
3. India has signed an agreement for its Participation in PISA 2021.
Which of the above statements is /are correct?
a. 1, 2 and 3
b. 1 and 3 only
c. 1 and 2 only
d. 1 only
Answer: B
Explanation: Statement 1 is correct.The Program for International Student Assessment
(PISA) is an international assessment that measures 15-year-old students’ reading,
mathematics, and science literacy every three years. First conducted in 2000, the major domain
of study rotates between reading, mathematics, and science in each cycle.
Statement 2 is incorrect.PISA is coordinated by the Organization for Economic Cooperation
and Development (OECD), an intergovernmental organization of industrialized countries, and is
conducted in the United States by NCES.
Statement 3 is correct.The Union Human Resources Development Ministry has signed an
agreement with Organisation for Economic Co-operation and Development (OECD) for India’s
Participation in Programme for International Student Assessment- PISA 2021.
Why this question?
Kendriya Vidyalayas, Navodaya Vidyalayas, and all schools, private and government, in
Chandigarh will participate in the 2021 test.

Source: http://pib.nic.in/newsite/PrintRelease.aspx?relid=187830

https://nces.ed.gov/surveys/pisa/faq.asp
Refer Pulse Magazine 34 ( Jan 27 - Feb 2)

41. Consider the following statements regarding Mission Innovation (MI).


1. Mission Innovation (MI) is a global initiative of 37 countries which includes India and the
European Commission
2. Its mission is to accelerate global clean energy innovation with the objective to make clean
energy widely affordable.
Which of the above statements is /are correct?
a. 1 only
b. 2 only
c. Both 1 and 2
d. Neither 1 nor 2

Answer: B
Explanation: Statement 1 is incorrect. Statement 2 is correct.
 Mission Innovation (MI) is a global initiative of 23 countries and the European Commission
(on behalf of the European Union) working to reinvigorate and accelerate global clean energy
innovation with the objective to make clean energy widely affordable.
 MI was announced at COP21 on November 30, 2015, as world leaders came together in Paris
to commit to ambitious efforts to combat climate change.

Source: http://mission-innovation.net/our-members/

https://www.google.com/amp/s/www.thehindubusinessline.com/news/science/a-shot-
in-the-arm-for-efforts-to-promote-newer-bio-fuels/article22864041.ece/amp/
42. With reference to International Whaling Commission, consider the following
statements.
1. The IWC is the global body charged with the conservation of whales and the management of
whaling.
2. Membership of the IWC is open to any country in the world not restricting to countries having
a coast.
3. Japan has announced its decision to withdraw from the International Whaling Commission
(IWC).
Which of the above statements is /are correct?
a. 1 and 2 only
b. 1 and 3 only
c. 3 only
d. 1, 2 and 3

Answer: D
Explanation: All the statements are correct.
 The International Whaling Commission is an Inter-governmental Organisation whose
purpose is the conservation of whales and the management of whaling.
 The legal framework of the IWC is the International Convention for the Regulation of
Whaling. This Convention was established in 1946, making it one of the first pieces of
international environmental legislation.
 All member countries of the IWC are signatories to this Convention. The IWC has a current
membership of 89 Governments from countries all over the world.
 Membership of the IWC is open to any country in the world that formally adheres to the 1946
Convention. India joined the commission in 1981.

Why this question?


Japan has announced its decision to withdraw from the International Whaling Commission
(IWC). The withdrawal would enable Japan to resume commercial whaling activities. Japan has
said that it would undertake commercial whaling from July 2019 limited to Japan’s territorial
waters and exclusive economic zones.

Source: https://www.japantimes.co.jp/news/2018/12/26/national/japan-formally-
announces-iwc-withdrawal-resume-commercial-whaling/#.XIAY9WThU0M

https://iwc.int/home

Refer Pulse Magazine 30 ( Dec 30- Jan 5)

43. Consider the following statements regarding Chilika lake.


1. Chilika Lake was the first Indian wetland of international importance under the Ramsar
Convention.
2. Currently, both Chilika lake and Loktak lake are placed in the Montreux record.
Which of the above statements is /are correct?
a. 1 only
b. 2 only
c. Both 1 and 2
d. Neither 1 nor 2

Answer: A
Explanation: Statement 1 is correct.
 Chilika Lake is the largest coastal lagoon in India and the second largest coastal lagoon in the
world.
 Chilika Lake was designated the first Indian wetland of international importance under the
Ramsar Convention.
Statement 2 is incorrect.
 Currently, two wetlands of India are in Montreux record viz. Keoladeo National Park,
Rajasthan and Loktak Lake, Manipur.
 Chilka lake was placed in the record but was later removed from it.
What is Montreux Record?
The Montreux Record is a register of wetland sites on the List of Wetlands of International
Importance where changes in ecological character have occurred, are occurring, or are likely to
occur as a result of technological developments, pollution or other human interference.

Why this question?


The two-day National Chilika Bird Festival was held at on the banks of Chilika lake in Odisha.

Source: https://www.google.com/amp/s/www.thehindu.com/news/national/other-
states/birdwatchers-flock-to-odishas-chilika-lake/article26105479.ece/amp/

Refer Pulse Magazine 30 ( Dec 30- Jan 5)

44. Consider the following pairs.


Treaty/Convention : Purpose
1. Brasilia Declaration : Visually-impaired persons
2. Marrakesh Treaty : Road Safety
3. New York Declaration: Refugees and Migrants
Which of the above is/are correctly matched?
a. 1 and 2 only
b. 1 and 3 only
c. 3 only
d. 1, 2 and 3

Answer: C
Explanation:
 Marrakesh Treaty – The WIPO-administered Marrakesh Treaty makes the production and
international transfer of specially-adapted books for people with blindness or visual
impairments easier. It does this by establishing a set of limitations and exceptions to traditional
copyright law.
 New york declaration – for Refugees and Migrants which expresses the political will of
world leaders to save lives, protect rights and share responsibility.
 Brasilia Declaration – on Road Safety
 BUNKER convention – It was adopted to ensure that adequate, prompt, and effective
compensation is available to persons who suffer damage caused by spills of oil, when carried as
fuel in ships’ bunkers.

45. Consider the following statements about National Anti-profiteering Authority (NAA).
1. The NAA is headed by the Finance Minister.
2. It ensures that the benefits of the reduction in GST rates are passed on to the ultimate
consumers.
Which of the above statements is/are correct?
a. 1 only
b. 2 only
c. Both 1 and 2
d. Neither 1 nor 2

Answer - b
Answer: Statement 1 is incorrect.National Anti-profiteering Authority is headed by a senior
officer of the level of Secretary to the Government of India. There will be four Technical
Members from the Centre and/or the States.
Statement 2 is correct.The National Anti-profiteering Authority (NAA) is the institutional
mechanism under GST law to check the unfair profit-making activities by the trading
community. The Authority’s core function is to ensure that the benefits of the reduction is GST
rates on goods and services made by GST Council and proportional change in the Input tax
credit passed on to the ultimate consumers.
How is the anti-profiteering mechanism under CGST act?

CGST Act, 2017 mandates a 3-tier structure for the investigation and adjudication of the
complaints regarding profiteering.
1. National Anti-profiteering Authority
2. Directorate General of Anti-profiteering
3. State-level Screening Committees and Standing Committee

Source : http://www.naa.gov.in/page.php?id=faqs

http://pib.nic.in/newsite/PrintRelease.aspx?relid=187337

46. Consider the following statements regarding Kisan Urja Surksha Utthan
Mahaabhiyan.
1. The main aim of this scheme is to provide the farmers with advanced technology to generate
power.
2. It aims to promote decentralized solar power production.
3. It is being implemented by the Ministry of Agriculture and Farmers Welfare.

Which of the above statements is /are correct?


a. 1 and 2 only
b. 2 and 3 only
c. 1 only
d. 1, 2 and 3

Answer: A
Explanation: Statement 1 and 2 are correct. Kisan Urja Suraksha evam Utthatan Mahabhiyan
(KUSUM) is a solar farming scheme to provide additional income and water security to farmers.
It envisages setting up solar power projects on the agriculture land and providing options to
farmers to sell additional power to grid.
Objectives of KUSUM
 Promote decentralized solar power production
 Reduce transmission losses.
 To support the financial health of DISCOMs by reducing the burden of subsidy to the
agriculture sector.
 To support States to meet the RPOs targets
 To promote energy efficiency and water conservation.
 Provide water security to farmers through provision of assured water sources through solar
water pumps – both off-grid and grid connected.
 To provide reliable power to utilise the irrigation potential created by state irrigation
departments.
 To fill the void in solar power production in the intermediate range between roof tops and
large parks.
Statement 3 is incorrect.The Ministry of New and Renewable Energy has formulated a Scheme
'Kisan Urja Suraksha evam Utthaan Mahabhiyan (KUSUM)'.

Source: https://www.google.com/amp/s/www.thehindu.com/business/agri-
business/govt-to-spend-48000-cr-to-help-farmers-go-solar/article22637365.ece/amp/

Refer Pulse Magazine 34 ( Jan 27 - Feb 2)

47. The park has sub-Himalayan belt geographical and ecological characteristics. Its whole area
comprises of hills, marshy depressions, riverine belts, grasslands and a large lake. It is the place
where Project Tiger was first launched in 1973.
The above description refers to which National Park?
a. Ranthambore National Park
b. Kanha Tiger Reserve
c. Tadoba-Andhari Tiger Reserve
d. Jim Corbett National Park

Answer:D
Explanation:Jim Corbett National Park is the oldest national park in India and was established
in 1936 as Hailey National Park to protect the endangered Bengal tiger. It is located in Nainital
district of Uttarakhand and was named after Jim Corbett who played a key role in its
establishment. The park was the first to come under the Project Tiger initiative.The park has
sub-Himalayan belt geographical and ecological characteristics. An ecotourism destination.
Project Tiger
 Aims at conserving India’s national animal i.e. Tiger.It was launched in 1973.
 The tiger reserves are constituted on a core/buffer strategy.
 The core areas have the legal status of a national park or a sanctuary, whereas the buffer or
peripheral areas are a mix of forest and non-forest land, managed as a multiple use area.
 The Project Tiger aims to foster an exclusive tiger agenda in the core areas of tiger reserves,
with an inclusive people oriented agenda in the buffer.
 It is a Centrally Sponsored Scheme of the Ministry of Environment, Forests and Climate
Change providing central assistance to the tiger States for tiger conservation in designated tiger
reserves.
 The National Tiger Conservation Authority (NTCA) is a statutory body of the Ministry, with
an overarching supervisory / coordination role, performing functions as provided in the
Wildlife (Protection) Act, 1972.

Source:https://en.wikipedia.org/wiki/Jim_Corbett_National_Park

http://www.moef.nic.in/division/introduction-18

48. North Korea is located between which of the following seas?


a. Korean bay and Yellow sea
b. Korean bay and Sea of Japan
c. Yellow sea and East China sea
d. East China Sea and Sea of Japan

Answer:B
Explanation:
49. Which of the following is not an antecedent river?
a. Sutlej
b. Kosi
c. Subanshiri
d. Son

Answer: (d)
Explanation: Antecedent streams are those which are originated prior to the upliftment
of land surface. In other words, antecedent streams antedate the upliftment of an
upland or mountain across which they have maintained their present courses through
continuous down-cutting of their valleys.
Examples of antecedent streams are found in almost all of the folded mountains of the
world. Many of the major Himalayan rivers are the examples of antecedent streams e.g.,
the Indus, the Sutlej, the Ganga, the Ghaghra, the Kali, the Gandak, the Kosi, the
Brahmaputra etc.
Like antecedent streams superimposed streams are also not adjusted to regional
geological structures and slope and thus are in sequent or anti-consequent streams.
Superimposed stream means a river which, flowing on a definite geological formation
and structure, has inherited the characteristics of its previous form developed on upper
geological formation of entirely different structural characteristics. Son is an example
of superimposed drainage system.
Source: India Physical Geography; NCERT Class XI; Chapter 3: Drainage System; pg.
24

50. Which of the following is not a reason for the evolution of the peninsular drainage
basin?
a. Subsidence of the western flank
b. Wider continental shelf on the eastern side
c. Upheaval of the Himalayas
d. The tilt of the peninsular block from northwest to southeast direction.
Answer: (b)
Explanation: three major geological events in the distant past have shaped the present
drainage system of Peninsular India:
i. The subsidence of the western flank of the peninsula leading to its submergence
below the sea during early tertiary period. This submergence is responsible for a wider
continental shelf on the western side of the peninsula in the Arabian Sea. Hence it is not
reason for the evolution of the drainage system. It has disturbed the symmetrical
plan of the river on either side of the original watershed.
ii. The upheaval of the Himalayas when the northern flank of the peninsular block was
subjected to subsidence and the consequent trough faulting.
iii. Slight tilting of the peninsular block from northwest to southeast direction gave the
orientation to the entire drainage system towards the Bay of Bengal.
Source: India Physical Geography; NCERT Class XI; Chapter 3: Drainage System; pg.
27

51. Arrange the following rivers from south to north:


1. Penneru
2. Bhima
3. Tungabadra
4. Manjra
Select the correct answer using the code given below
a. 1-2-3-4
b. 1-3-2-4
c. 3-1-2-4
d. 3-2-1-4
Answer: (b)
Source: Atlas

52. Arrange the following tributaries of Yamuna from west to east:


1. Chambal
2. Betwa
3. Sind
4. Ken
Select the correct answer using the code given below
a. 1-3-2-4
b. 2-3-1-4
c. 1-3-4-2
d. 2-1-3-4

Answer: (a)

Explanation:
Source: Atlas

53. Which of the following rivers flow towards the west?


1. Shetrunji
2. Mahi
3. Kalinadi
4. Penner
Select the correct answer using the code given below
a. 1, 2 and 3
b. 2, 3 and 4
c. 1, 2 and 4
d. All of the above

Answer: (a)
Explanation: the rivers flowing through the Arabian Sea has short course. Shetrunji rises
near Dalkahwa in Amerli district.
The Mahi river is flows into the gulf of Kambhat. The Kalinadi rises near Belgaum
district and falls into the Karwar bay. The Pennar is an east flowing river.
Source: India Physical Geography; NCERT Class XI; Chapter 3: Drainage System; pg. 28

54. Consider the following statements:


1. Peninsular rivers are older than all Himalayan rivers.
2. Super imposed drainage pattern is mostly found in peninsular rivers.
3. Most of the peninsular rivers flow in rift valley.
Which of the above statements is/are correct?
a. 1 and 2 only
b. 2 only
c. 1 and 3 only
d. 3 only
Answer: (b)
Explanation: Peninsular Rivers are older than the Himalayan Rivers in general due to
different geological time period in their formation. However, there are some antecedent
rivers like Sutlej and Indus which are antecedent rivers. Hence statement 1 is incorrect.
The peninsular drainage system is a super-imposed pattern rejuvenated resulting i
trellis, radial and rectangular patterns. Whereas the Himalayan rivers have antecedent
and consequent drainage leading to dendritic pattern.
Statement 3 is incorrect. only river Narmada and Tapi flow in rift valleys in the
peninsula. The other rivers are east flowing rivers which forms delta before joining the
Bay of Bengal.
Source: India Physical Geography; NCERT Class XI; Chapter 3: Drainage System; pg.
29

55. Consider the following Statements:


1. Indian Climate is affected by factors like the Latitude and Altitude of a Place.
2. India experiences a predominantly hot monsoonal type of climate.
Which of the above Statements is/are correct?
a. 1 only
b. 2 only
c. Both 1 and 2
d. Neither 1 and 2

Answer: (c)

Explanation: Statement 1 is correct. India’s climate is controlled by a number of


factors related to location and relief and factors related to air pressure and winds.
Northern part of the India lies in sub-tropical and temperate zone and the part lying
south of the Tropic of Cancer falls in the tropical zone. The tropical zone being nearer to
the equator, experiences high temperatures throughout the year with small daily and
annual range. Altitude- Temperature decreases with height. Due to thin air, places in the
mountains are cooler than places on the plains.

Statement 2 is correct. Monsoon connotes the climate associated with seasonal


reversal in the direction of winds. India has hot monsoonal climate which is the
prevalent climate in south and south-east Asia. The climate of India has many regional
variations expressed in the pattern of winds, temperature and rainfall, rhythm of
seasons and the degree of wetness or dryness. These regional diversities may be
described as sub-types of monsoon climate.

Source: India: Physical Environment, NCERT, Class XI, Chapter 4-Climate, Pgs-33-34.

56. Consider the Following Statements:


1. The Inter Tropical Convergence Zone (ITCZ) is a low pressure zone located at the
equator where trade winds converge, and so, it is a zone where air tends to descend.
2. By July, The ITCZ is located 20°-25° North of the Equator. Similarly, In Winter it
moves southward.
Which of the above Statements is/are correct?
a. 1 only
b. 2 only
c. Both 1 and 2
d. Neither 1 and 2

Answer: (b)
Explanation: The Inter Tropical Convergence Zone (ITCZ) is a low pressure zone located
at the equator where trade winds converge, and so, it is a zone where air tends to
ascend. In July, the ITCZ is located around 20°N-25°N latitudes (over the Gangetic plain),
sometimes called the monsoon trough. This monsoon trough encourages the
development of thermal low over north and northwest India. Due to the shift of ITCZ,
the trade winds of the southern hemisphere cross the equator between 40° and 60°E
longitudes and start blowing from southwest to northeast due to the Coriolis force. It
becomes southwest monsoon. In winter, the ITCZ moves southward, and so the reversal
of winds from northeast to south and southwest, takes place.

Source: India: Physical Environment, NCERT, Class XI, Chapter 4-Climate, Pgs-37.

57. Consider the Following:


1. Latitudinal location.
2. Distance from Sea
3. Coriolis Force
4. Surface Winds and Air masses
5. Phenomena like Tropical depressions.
Which of the above directly affects Climate of a region?
a. 1,2,3 and 4
b. 1,2 ,4 and 5
c. 1,2 and 4
d. All of the above

Answer: (b)
Explanation: India’s climate is controlled by a number of factors which can be broadly
divided into two groups — factors related to location and relief, and factors related to
air pressure and winds. Latitude and altitude of a region, Along with its distance from
water body influence the climatic conditions. Tropical depressions and western
disturbances affect the periodic changes in a region. Coriolis force does not directly
impact a region’s climate.
Source: India: Physical Environment, NCERT, Class XI, Chapter 4-Climate, Pgs-37

58. Consider the Following Statements:


1. Jet Streams are upper air circulations moving at high speeds, at an altitude of 9-13
Km from east to west.
2. The Atmospheric Pressure Variations at surface affect these Jet streams considerably.
Which of the above Statements is/are correct?
a. 1 only
b. 2 only
c. Both 1 and 2
d. Neither 1 and 2

Answer: (d)
Explanation: Higher up in the lower troposphere, about three km above the surface of
the earth, a different pattern of air circulation is observed. The variations in the
atmospheric pressure closer to the surface of the earth have no role to play in the
making of air circulation. All of Western and Central Asia remains under the influence of
westerly winds along the altitude of 9-13 km from west to east. These winds blow
across the Asian continent at latitudes north of the Himalayas roughly parallel to the
Tibetan highlands. These are known as jet streams.
Source:
India: Physical Environment, NCERT, Class XI, Chapter 4-Climate, Pgs-35

59. Consider the Following:


1. The Westerly jet streams play a crucial role in preventing the western cyclonic
disturbances from entering Indian subcontinent in the winter.
2. The Easterly jet streams helps in the distribution of monsoon rainfall in India.
Which of the above statements is/are incorrect?
a. 1 only
b. 2 only
c. Both 1 and 2
d. Neither 1 and 2

Answer: (a)
Explanation: The western cyclonic disturbances which enter the Indian subcontinent
from the west and the northwest during the winter months, originate over the
Mediterranean Sea and are brought into India by the westerly jet stream. An increase in
the prevailing night temperature generally indicates an advance in the arrival of these
cyclones disturbances. Hence Statement 1 is Incorrect.
The easterly jet stream steers the tropical depressions into India. These depressions
play a significant role in the distribution of monsoon rainfall over the Indian
subcontinent. The tracks of these depressions are the areas of highest rainfall in India.
The frequency at which these depressions visit India, their direction and intensity, all go
a long way in determining the rainfall pattern during the southwest monsoon period.
Statement 2 is correct.
Source: India: Physical Environment, NCERT, Class XI, Chapter 4-Climate, Pgs-36-37.

60. Consider the Following:


1. When the South-west Monsoon sets in, the northwestern part of India experiences a
low pressure centre, in conjunction with a high pressure zone over the southern Indian
Ocean.
2. The withdrawing Westerly jet stream also affects the position of ICTZ.
Which of the above statements is/are correct?
a. 1 only
b. 2 only
c. Both 1 and 2
d. Neither 1 and 2

Answer: (c)
Explanation: The differential heating of land and sea during the summer months is the
mechanism which sets the stage for the monsoon winds to drift towards the
subcontinent. During April and May when the sun shines vertically over the Tropic of
Cancer, the large landmass in the north of Indian ocean gets intensely heated. This
causes the formation of an intense low pressure in the north-western part of the
subcontinent. Since the pressure in the Indian Ocean in the south of the landmass is high
as water gets heated slowly, the low pressure cell attracts the southeast trades across
the Equator. These conditions help in the northward shift in the position of the ITCZ.
The southwest monsoon may thus, be seen as a continuation of the southeast trades
deflected towards the Indian subcontinent after crossing the Equator .Hence Statement
1 is correct.

The shift in the position of the ITCZ is also related to the phenomenon of the withdrawal
of the westerly jet stream from its position over the north Indian plain, south of the
Himalayas. The easterly jet stream sets in along 15°N latitude only after the western jet
stream has withdrawn itself from the region. This easterly jet stream is held responsible
for the burst of the monsoon in India. Statement 2 is correct.
Source: India: Physical Environment, NCERT, Class XI, Chapter 4-Climate, Pgs-37-38.
61. Consider the Following Statements about El-Niño:
1. El-Niño is a weather system involving oceanic and atmospheric phenomena with the
appearance of warm currents off the coast of Peru in the Eastern Pacific.
2. EI-Nino is used in India for forecasting long range monsoon rainfall, as it is a regular
phenomenon accompanying Indian Monsoon.
Which of the above statements is/are correct?
a. 1 only
b. 2 only
c. Both 1 and 2
d. Neither 1 and 2

Answer: (a)
Explanation: EI-Nino is a complex weather system that appears once every three to
seven years, bringing drought, floods and other weather extremes to different parts of
the world. The system involves oceanic and atmospheric phenomena with the
appearance of warm currents off the coast of Peru in the Eastern Pacific and affects
weather in many places including India. Statement 1 is correct.
EI-Nino is used in India for forecasting long range monsoon rainfall. In 1990-91, there
was a wild EI-Nino event and the onset of southwest monsoon was delayed over most
parts of the country ranging from five to twelve days.El-Nino is an extreme weather
phenomenon which occurs periodically.It is not a regular feature of Indian monsoon.
Hence, Statement 2 is incorrect.
Source:
India: Physical Environment, NCERT, Class XI, Chapter 4-Climate, Pgs-38

62. Consider the Following;


1. The Phenomenon of El-Niño affects Fish and plankton life in the water and causes
distortion in the evaporation of sea water.
2. A strong El-Niño is beneficial for the monsoon rainfall in India.
Which of the above statements is/are incorrect?
a. 1 only
b. 2 only
c. Both 1 and 2
d. Neither 1 and 2

Answer: (b)
Explanation: EI-Nino is merely an extension of the warm equatorial current which gets
replaced temporarily by cold Peruvian current or Humboldt current. This current
increases the temperature of water on the Peruvian coast by 10°C. This results in:
(i) the distortion of equatorial atmospheric circulation;
(ii) irregularities in the evaporation of sea water;
(iii) reduction in the amount of planktons which further reduces the number of fish in
the sea.
Statement1 is correct.
A Strong El-Nino would result in a Colder currents on the western pacific ocean which
would weaken the high pressure centres of the indian oceans. The monsoon winds
would not be strong and Rainfall not adequate. Statement 2 is incorrect.
Source:
India: Physical Environment, NCERT, Class XI, Chapter 4-Climate, Pgs-38

63. Consider the Following matches:


1. Mango showers- Kerala and Karnataka
2. Nor westers- Bengal
3. Blossom showers – Punjab
Which of the above is/are correctly matched?
a. 1 only
b. 1 and 2
c. 1 and 3
d. All of the above

Answer: (b)
Explanation:
Mango Shower : Towards the end of summer, there are pre-monsoon showers which are
a common phenomena in Kerala and coastal areas of Karnataka. Locally, they are known
as mango showers since they help in the early ripening of mangoes.
Blossom Shower : With this shower, coffee flowers blossom in Kerala and nearby areas.
Nor Westers : These are dreaded evening thunderstorms in Bengal and Assam. Their
notorious nature can be understood from the local nomenclature of ‘Kalbaisakhi’, a
calamity of the month of Baisakh. These showers are useful for tea, jute and rice
cultivation. In Assam, these storms are known as “Bardoli Chheerha”.
Source:
India: Physical Environment, NCERT, Class XI, Chapter 4-Climate, Pgs-45

64. Consider the following statements:


1. The composition of different materials like minerals, water, humus and air in the soil
is known as soil profile.
2. Soil survey of India and ICAR has classified the Indian soils as per the USDA’s soil
taxonomy.
Which of the above statements is/are correct?
a. 1 only
b. 2 only
c. Both 1 and 2
d. Neither 1 nor 2

Answer: (b)
Explanation: statement 1 is incorrect. The arrangement of soil in layers beneath the
earth is called soil profile. A soil profile consists of four layers of soil called horizons.
The O horizon, A horizon, B horizon and C horizon are composed of different soil
materials and each goes through different processes. From decomposing organic
material to unconsolidated bedrock, each soil layer is different and is important to the
stability of soil and vegetation growth. Each layer gets longer as you go farther down the
list, starting with a 2-inch layer of composition and ending with a 48-inch layer of
bedrock.
Statement 2 is correct. The Soil Survey of India and the Indian Council of Agricultural
Research studies the soils of India and classified it on the basis of their nature and
Character as per the United States Department of Agriculture (USDA) soil taxonomy.
There are 8 categories ranging from Inceptisols to Mollisols.
Source: India: Physical Environment, NCERT, Class XI, Chapter 6: Soils, Pg. 68-69

65. When you travel in certain parts of India, you will notice red soil. What is the
main reason for this red colour?
a. Abundance of Magnesium
b. Accumulated humus
c. Presence of ferric oxide
d. Abundance of phosphates

Answer: (c)
Explanation: red soil develops in crystalline igneous rocks in areas of low rainfall
mostly in the eastern and southern part of Deccan Plateau. The soil develops a reddish
colour due to wide diffusion of iron in crystalline and metamorphic rocks. It becomes
yellow when it is hydrated.
Source: India: Physical Environment, NCERT, Class XI, Chapter 6: Soils, Pg. 7, UPSC 2000.

66. Consider the following statements regarding red soil:


1. Red soil is deficient in nitrogen but rich in Potash.
2. They are both fertile and poor in fertility based on the region where they are found.
3. They are mostly found in Odisha, Chattisgarh and Tamil Nadu.
Which of the above statements is/are correct?
a. 1 and 2 only
b. 1 and 3 only
c. 2 and 3 only
d. 1, 2 and 3

Answer: (d)
Explanation: the red soil is formed by the decomposition of ancient crystalline rocks like
granite and gneiss and from rocks rich in iron and magnesium. However they are poor
in nitrogen, phosphates and humus. It is rich in potash. The finger-grained red and
yellow soil is normally fertile, whereas coarse grained soils found in dry upland areas
are poor in fertility. These soils are found in the eastern and southern part of Deccan
plateau like Tamil Nadu and Kerala. Majorly found in Odisha, Chattisgarh. Hence all the
statements are correct.
Source: India: Physical Environment, NCERT, Class XI, Chapter 6: Soils, Pg. 71

67. Consider the following statements:


1. Alluvial soil covers the largest part of India.
2. Alluvial soil is poor in potash and rich in phosphorous.
3. Both the Khadar and Bhangar soils contain calcareous concretions.
Which of the above statements is/are correct?
a. 1 and 2 only
b. 2 and 3 only
c. 1 and 3 only
d. 1, 2 and 3
Answer: (c)
Explanation: the alluvial soil covers 40% of the total area of the country. Hence
statement 1 is correct. They are depositional soils transported and deposited by rivers
and streams. They are largely found in the gangetic plains, corridors Of Rajasthan,
Gujarat and the deltas of east coast. They vary in nature from sandy to loamy. They are
generally rich in potash and poor in phosphorous. Hence statement 2 is incorrect.
There are two different types of alluvium. Khadar is the new alluvium and is deposited
by floods annually enriching the soil. Bhangar represents a system of old alluvium
deposited away from the floodplains. Both the alluvium contains calcareous concretions
(Kanker). The content in the soil decreases from west to east in the floodplains. Hence
statement 3 is correct.
Source: India: Physical Environment, NCERT, Class XI, Chapter 6: Soils, Pg. 69

68. Which one of the following is called ‘Regur soil’?


a. Red soil
b. Black soil
c. Peaty soil
d. Laterite soil

Answer: (b)
Explanation: the black soil is called regur soil or black cotton soil. It is very deep and
found in the western part of Deccan plateau. It is also found in some parts of Andhra
Pradesh, Tamil Nadu etc. this black soil is generally clayey, deep and impermeable. They
are like self ploughing as they swell and become sticky when wet and shrink when
dried.
Source: India: Physical Environment, NCERT, Class XI, Chapter 6: Soils, Pg. 71

69. Consider the following statements regarding saline soil:


1. Saline soil is poor in calcium content.
2. It is highly infertile.
3. Gypsum is a mineral which reduces salinity.
Which of the above statements is/are correct?
a. 1 and 2 only
b. 2 and 3 only
c. 1 and 3 only
d. 1, 2 and 3

Answer: (d)
Explanation: Saline soils contain large portions of sodium, potassium and magnesium.
They are poor in nitrogen and calcium. Hence it is highly infertile. These soils mostly
form in dry arid areas and in waterlogged swampy areas. In areas of intensive
cultivation due to excessive irrigation the fertile soil is becoming saline due to capillary
action that is the salts from below is found in the surface. To solve this problem it is
advised to add gypsum as it reduces the salinity in the soil. Hence all statements
are correct.
Source: India: Physical Environment, NCERT, Class XI, Chapter 6: Soils, Pg. 72.

70. In India, the problem of soil erosion is associated with which of the following?
1. Terrace cultivation
2. Deforestation
3. Tropical climate
Select the correct answer using the code given below:
a. 1 and 2 only
b. 2 only
c. 1 and 3
d. 1, 2 and 3

Answer: (b)
Explanation: Terrace cultivation is a method of soil conservation. Terraces are graded
in a way as to eliminate finger gullies etc. hence it is not a reason for soil erosion.
Deforestation contributes largely to soil erosion as loss of vegetation cover exposes the
soil to easy erosion. Other factors for soil erosion are water erosion, excessive leaching,
wind erosion etc. Tropical climate is generally subjected to high rainfall and vegetative
growth. Hence is not a factor as such for soil erosion.

71. Consider the following pairs


1. Kerala - Tapioca
2. Maharashtra - Cotton
3. West Bengal - Jute
4. Gujarat - Groundnut
Which of the above pairs of states and their important crops are correctly
matched?
a. 1, 2 and 3
b. 1, 2 and 4
c. 1, 3 and 4
d. 2, 3 and 4

Answer: (d)
Explanation: Kerala is a tropical climatic region with heavy rainfall. The major crops
here are coconut, rubber, spices, tea etc. Tapioca is still a major crop in lower
latitudes which is in the equatorial region.
Cotton is widely grown in the black soil rich region in Maharashtra. West Bengal is
again a delta region with heavy rainfall conducive to jute cultivation. It is the largest
producer of jute. Groundnut is grown in Gujarat, Andhra Pradesh etc.
Source: Maps of India

72. Consider the following statements:


1. Jowar is both a kharif and Rabi crop.
2. Jowar can be grown in all types of soil texture ranging from sandy to loamy.
3. Jowar can be grown in regions with temperature as high as 45’c with rainfall of 25cm
average.
Which of the above statements is/are correct?
a. 1 and 2 only
b. 2 and 3 only
c. 1 and 3 only
d. 1, 2 and 3
Answer: (a)
Jowar also known as Grain sorghum can be grown on many different soils.
Sorghum will yield best on deep, fertile, well-drained loamy soils. However, it is quite
tolerant of shallow soil and droughty conditions. Jowar can be grown successfully on
clay, clay loam, or sandy loam soils. Sorghum requires warm conditions but it can be
grown under a wide range of conditions.
It is also widely grown in temperate regions and at altitudes of up to 2300 m in the
tropics. It can tolerate high temperature throughout its life cycle better than any other
crop. Sorghum requires about 26-30oC temperature for good growth. The minimum
temperature for the germination of the sorghum seed is 7 to 10oC.
Grain sorghum does not germinate and grow well under cool soil conditions. Poor
emergence and seedling growth may result if planted before soil temperatures reach
35oC. Sorghum is best adapted to areas having an average annual rainfall between 45 to
65 cm (17 to 25 inches).
Source: agropedia

73. Tank irrigation is practised mainly in peninsular India because:


1. Undulating relief and hard rocks make it difficult to dig canal and wells.
2. Rivers are rain-fed.
3. Compact nature of population and agricultural fields.
Select the correct answer using the codes given below:
a. 1 and 2
b. 2 and 3
c. 1 and 3
d. 1, 2 and 3

Answer: a
Explanation: different types of irrigation are followed in different parts of the country
according to the geo-climatic and physiographic feature of the area. In the peninsular
India which is made up of plateau and hard rocks tank irrigation is mostly suitable.
Hard rocks make it difficult for canal construction an also the rivers here are
seasonal. Rain fed rivers hence storage in tanks is more suitable. In northern parts of
India, canal irrigation is more suitable. Hence statement 1 and 2 is correct. Statement 3
is incorrect. The type of population settlement is not a reason for the type of irrigation
followed.
Source: CDS 2009

74. Tamil Nadu is a leading producer of mill-made cotton yarn in the country.
What could be the reason?
1. Black cotton soil is the predominant soil type in the state.
2. Rich pool of skilled labour is available.
Which of the statements given above is/are correct?
a. 1 only
b. 2 only
c. Both 1 and 2
d. Neither 1 nor 2

Answer: (b)
Explanation: black soil is present in Tamil Nadu but is not the predominant soil type.
Red soil is the predominant type of soil in the state. Tamil Nadu is one of the major
states where cotton mills are largely spread. Availability of skilled labour is one of the
major reasons for it to be the leading producer of cotton mill-made yarn in the country.
Hence statement 2 is correct.
Source: UPSC 2000

75. The alpine vegetation in the western Himalayas is found only up to a height of
3000 mts while in eastern Himalayas is found up to a height of 4000 mts. The
reason for this variation is:
a. Eastern Himalayas are higher than western Himalayas.
b. Eastern Himalayas are steeper than the western Himalayas.
c. Eastern Himalayas get more monsoon rainfall than western Himalayas.
d. Eastern Himalayan rocks are more fertile than the western Himalayan rocks.

Answer: (c)
Explanation: temperature and precipitation are the foremost factors which determine
the nature of vegetation in a particular place. The physiographic location is the factor
which influences the pattern of temperature and precipitation. Hence the likelihood of
higher rainfall is higher in eastern Himalayas than in western Himalayas. The alpine
vegetation is found in higher altitudes in eastern Himalayas than in western Himalayas.

76. Consider the following statements about United Nations Convention to


Combat Desertification (UNCCD):
1. It is a legally binding international agreement linking environment and development
to sustainable land management.
2. It addresses specifically the arid, semi-arid and dry sub-humid areas.
3. UNCCD is committed to a centralised approach to combat land degradation.
4. The Conference of Parties (COP) was established by the Convention as the supreme
decision-making body which meets biannually.
Which of the statements given above is/are correct?
(a) 1 and 2
(b) 2 only
(c) 1, 2 and 4
(d) 1, 3 and 4

Solution: (a)
Established in 1994, the United Nations Convention to Combat Desertification (UNCCD)
is the sole legally binding international agreement linking environment and
development to sustainable land management. Hence Statement 1 is correct. The
Convention addresses specifically the arid, semi-arid and dry sub-humid areas, known
as the drylands, where some of the most vulnerable ecosystems and peoples can be
found. Hence Statement 2 is correct.
The Convention’s 197 parties work together to improve the living conditions for people
in drylands, to maintain and restore land and soil productivity, and to mitigate the
effects of drought. The UNCCD is particularly committed to a bottom-up approach,
encouraging the participation of local people in combating desertification and land
degradation. Hence Statement 3 is incorrect. The UNCCD secretariat facilitates
cooperation between developed and developing countries, particularly around
knowledge and technology transfer for sustainable land management.
As the dynamics of land, climate and biodiversity are intimately connected, the UNCCD
collaborates closely with the other two Rio Conventions; the Convention on Biological
Diversity (CBD) and the United Nations Framework Convention on Climate Change
(UNFCCC), to meet these complex challenges with an integrated approach and the best
possible use of natural resources.
The Conference of Parties (COP) was established by the Convention as the supreme
decision-making body; it comprises ratifying governments and regional economic
integration organizations, such as the European Union. Up to today, the COP had held
thirteen sessions; it has been meeting biennially since 2001. COP13 took place in 2017
in Ordos, China. Hence Statement 4 is incorrect.

77. Consider the following statements about TRAFFIC (The Wildlife Trade
Monitoring Network):
1. TRAFFIC is an inter-governmental organisation working globally on trade in wild
animals and plants.
2. It is a joint conservation programme of WWF and IUCN.
3. It actively collaborates with the CITES Secretariat.
Which of the above statements is/are correct?
(a) 2 only
(b) 2 and 3 only
(c) 1 and 2 only
(d) 1, 2 and 3

Solution: (b)
TRAFFIC is a leading non-governmental organisation working globally on trade in wild
animals and plants in the context of both biodiversity conservation and sustainable
development. Hence Statement 1 is incorrect.
It is a joint conservation programme of WWF and IUCN. Hence Statement 2 is correct.
It was established in 1976 by the Species Survival Commission of IUCN principally as a
response to the entry into force during the previous year of CITES. It undertakes its
activities in close collaboration with governments and the CITES Secretariat. Hence
Statement 3 is correct.

78. Consider the following statements regarding Ramsar Convention:


1. Wise use of wetlands refers to the maintenance of ecological character of wetland
through the implementation of ecosystem approaches.
2. Its implementation involves adopting integrated management plans at wetland sites.
3. Contracting Parties of the Convention commit to work towards the wise use of the
wetlands and water resources in their territory.
Which of the statements given above is/are incorrect?
(a) 1 only
(b) 1 and 2 only
(c) 2 and 3 only
(d) None

Solution: (d)
At the centre of the Ramsar philosophy is the “wise use” of wetlands. When they accede
to the Convention, Contracting Parties commit to work towards the wise use of all the
wetlands and water resources in their territory, through national plans, policies and
legislation, management actions and public education. Hence Statement 3 is correct.
The Convention defines wise use of wetlands as “the maintenance of their ecological
character, achieved through the implementation of ecosystem approaches, within the
context of sustainable development”. Hence Statement 1 is correct. Wise use can thus
be seen as the conservation and sustainable use of wetlands and all the services they
provide, for the benefit of people and nature.
In 1990 the Contracting Parties adopted Guidelines for the implementation of the wise
use concept. The Guidelines emphasized the importance of:
1. adopting national wetland policies, either separately or as a component of wider
initiatives such as national environmental action plans;
2. developing programmes covering wetland inventory, monitoring, research, training,
education and public awareness;
3. developing integrated management plans at wetland sites. Hence Statement 2 is
correct.

79. Consider the following statements about Paris Agreement:


1. It aims to keep global temperature rise this century well below 2 degrees Celsius
above pre-industrial levels.
2. Appropriate financial flows are envisaged to support action by developing countries
and the most vulnerable countries.
Which of the statements given above is/are correct?
(a) 1 only
(b) 2 only
(c) Both 1 and 2
(d) Neither 1 nor 2

Solution: (c)
The 2015 Paris Agreement, adopted in Paris on 12 December 2015, marks the latest
step in the evolution of the UN climate change regime and builds on the work
undertaken under the Convention. The Paris Agreement charts a new course in the
global effort to combat climate change. The Paris Agreement seeks to accelerate and
intensify the actions and investment needed for a sustainable low carbon future. Its
central aim is to strengthen the global response to the threat of climate change by
keeping a global temperature rise this century well below 2 degrees Celsius above pre-
industrial levels and to pursue efforts to limit the temperature increase even further to
1.5 degrees Celsius. Hence Statement 1 is correct. The Agreement also aims to
strengthen the ability of countries to deal with the impacts of climate change.
To reach these ambitious goals, appropriate financial flows, including by, before 2025,
setting a new goal on the provision of finance from the USD 100 billion floor, and an
enhanced capacity building framework, including an Initiative for Capacity Building, will
be put in place: thus supporting action by developing countries and the most vulnerable
countries, in line with their own national objectives. The Agreement will also enhance
transparency of action and support through a more robust transparency framework.
Hence Statement 2 is correct.
80. The International Consortium on Combating Wildlife Crime (ICCWC) was
established during:
(a) UNFCCC CoP-24
(b) International Tiger Forum
(c) CITES CoP-24
(d) CITES CoP-16

Solution: (b)
The International Consortium on Combating Wildlife Crime (ICCWC) is the collaborative
effort of five inter-governmental organizations working to bring coordinated support to
the national wildlife law enforcement agencies and to the sub-regional and regional
networks that, on a daily basis, act in defense of natural resources.
The ICCWC partners are the Convention on International Trade in Endangered Species
of Wild Fauna and Flora (CITES) Secretariat, INTERPOL, the United Nations Office on
Drugs and Crime, the World Bank and the World Customs Organization. This powerful
alliance was formally established on 23 November 2010 in St. Petersburg, Russia during
the International Tiger Forum when the signatures of all partners were included on the
Letter of Understanding.

ICCWC’s mission is to strengthen criminal justice systems and provide coordinated


support at national, regional and international level to combat wildlife and forest crime
to ensure perpetrators of serious wildlife and forest crime will face a formidable and
coordinated response.

81. Consider the following statements about CITES (the Convention on


International Trade in Endangered Species of Wild Fauna and Flora):
1. CITES is legally binding on the Parties to the Convention.
2. The CITES Secretariat is administered by IUCN
Which of the statements given above is/are correct?
(a) 1 only
(b) 2 only
(c) Both 1 and 2
(d) Neither 1 nor 2

Solution: (a)
CITES (the Convention on International Trade in Endangered Species of Wild Fauna and
Flora) is an international agreement between governments. Its aim is to ensure that
international trade in specimens of wild animals and plants does not threaten their
survival. CITES was signed in Washington D.C. on 3 March 1973 and entered into force
on 1 July 1975.
CITES is an international agreement to which States and regional economic integration
organizations adhere voluntarily. States that have agreed to be bound by the
Convention are known as Parties. Although CITES is legally binding on the Parties – in
other words they have to implement the Convention – it does not take the place of
national laws. Hence Statement 1 is correct. Rather it provides a framework to be
respected by each Party, which has to adopt its own domestic legislation to ensure that
CITES is implemented at the national level.
The CITES Secretariat is administered by UNEP and is located at Geneva, Switzerland. It
has a pivotal role, fundamental to the Convention. Hence Statement 2 is incorrect.

82. Consider the following statements:


1. UNCCD 2018-2030 Strategic Framework is a comprehensive global commitment to
achieve Land Degradation Neutrality (LDN).
2. It will help reduce the impacts of drought on vulnerable populations.
3. Green Climate Fund is the financial mechanism of the UNCCD.
Which of the statements given above is/are correct?
(a) 1 only
(b) 1 and 2
(c) 3 only
(d) 1, 2 and 3

Solution: (b)
The new UNCCD 2018-2030 Strategic Framework adopted at UNCCD COP13 in Ordos,
China, is the most comprehensive global commitment to achieve Land Degradation
Neutrality (LDN) in order to restore the productivity of vast expanses of degraded land,
improve the livelihoods of more than 1.3 billion people, and reduce the impacts of
drought on vulnerable populations to build a future that avoids, minimizes, and
reverses desertification/land degradation and mitigates the effects of drought in
affected areas at all levels. Hence Statement 1 and 2 are correct.
The Global Environment Facility (GEF) became a financial mechanism of the UNCCD in
May 2010, when the GEF instrument was amended by its Fourth Assembly. Hence
Statement 3 is incorrect.

83. Consider the following statements about International Tropical Timber


Organization (ITTO):
1. It is an intergovernmental organization promoting the sustainable management and
conservation of tropical forests.
2. All countries which have presence of tropical forests are a part of this organisation.
Which of the statements given above is/are correct?
(a) 1 only
(b) 2 only
(c) Both 1 and 2
(d) Neither 1 nor 2

Solution: (a)
The International Tropical Timber Organization (ITTO) is an intergovernmental
organization promoting the sustainable management and conservation of tropical
forests and the expansion and diversification of international trade in tropical timber
from sustainably managed and legally harvested forests. Hence Statement 1 is correct.
ITTO:
 Develops internationally agreed policy guidelines and norms to encourage
sustainable forest management (SFM) and sustainable tropical timber industries and
trade.
 Assists tropical member countries to adapt such guidelines and norms to local
circumstances and to implement them in the field through projects and other activities.
 Collects, analyzes and disseminates data on the production and trade of tropical
timber.
 Promotes sustainable tropical timber supply chains.
 Helps develop capacity in tropical forestry.
ITTO’s membership represents about 90% of the global tropical timber trade and more
than 80% of the world’s tropical forests. Hence Statement 2 is incorrect.

84. Consider the following statements about Bonn Challenge:


1. It is a global effort to bring 350 million hectares of the world’s deforested and
degraded land into restoration by 2020.
2. It has a forest landscape restoration (FLR) approach, which aims to restore ecological
integrity at the same time as improving human well-being
3. It could bring direct additional income opportunities for rural communities.
Which of the statements given above is/are correct?
(a) 1 only
(b) 1 and 2
(c) 3 only
(d) 2 and 3

Solution: (d)
The Bonn Challenge was launched in 2011 by the Government of Germany and IUCN,
and later endorsed and extended by the New York Declaration on Forests at the 2014
UN Climate Summit. It is a global effort to bring 150 million hectares of the world’s
deforested and degraded land into restoration by 2020, and 350 million hectares by
2030. Hence Statement 1 is incorrect.
Underlying the Bonn Challenge is the forest landscape restoration (FLR) approach,
which aims to restore ecological integrity at the same time as improving human well-
being through multifunctional landscapes. Hence Statement 2 is correct.
The restoration of 150 million hectares of degraded and deforested lands in biomes
around the world could bring direct additional income opportunities for rural
communities. Hence Statement 3 is correct. Achieving the 350 million hectare goal
will result in benefits from watershed protection, improved crop yields and forest
products, and could sequester up to 1.7 gigatonnes of carbon dioxide equivalent
annually.
The Bonn Challenge is not a new global commitment but rather a practical means of
realizing many existing international commitments, including the CBD Aichi Target 15,
the UNFCCC REDD+ goal, and the Rio+20 land degradation neutrality goal. It is an
implementation vehicle for national priorities such as water and food security and rural
development while contributing to the achievement of international climate change,
biodiversity and land degradation commitments.

85. Consider the following:


1. If it is a site containing representative, rare or unique wetland types
2. If it regularly supports 50,000 or more waterbirds.
3. If it is an important source of food for fishes,
Which of these are criteria for identifying Wetlands of International Importance?
(a) 1 only
(b) 2 and 3 only
(c) 1 and 3 only
(d) 1, 2 and 3

Solution: (c)
The nine criteria for identifying Wetlands of International Importance
Group A of the Criteria. Sites containing representative, rare or unique wetland types
Criterion 1: A wetland should be considered internationally important if it contains a
representative, rare, or unique example of a natural or near-natural wetland type found
within the appropriate biogeographic region.
Hence Statement 1 is correct.
Group B of the Criteria. Sites of international importance for conserving biological
diversity
Criteria based on species and ecological communities
Criterion 2: A wetland should be considered internationally important if it supports
vulnerable, endangered, or critically endangered species or threatened ecological
communities.
Criterion 3: A wetland should be considered internationally important if it supports
populations of plant and/or animal species important for maintaining the biological
diversity of a particular biogeographic region.
Criterion 4: A wetland should be considered internationally important if it supports
plant and/or animal species at a critical stage in their life cycles, or provides refuge
during adverse conditions.
Specific criteria based on waterbirds
Criterion 5: A wetland should be considered internationally important if it regularly
supports 20,000 or more waterbirds. Hence Statement 2 is incorrect.
Criterion 6: A wetland should be considered internationally important if it regularly
supports 1% of the individuals in a population of one species or subspecies of
waterbird.
Specific criteria based on fish
Criterion 7: A wetland should be considered internationally important if it supports a
significant proportion of indigenous fish subspecies, species or families, life-history
stages, species interactions and/or populations that are representative of wetland
benefits and/or values and thereby contributes to global biological diversity.
Criterion 8: A wetland should be considered internationally important if it is an
important source of food for fishes, spawning ground, nursery and/or migration path on
which fish stocks, either within the wetland or elsewhere, depend. Hence Statement 3
is correct.
Specific criteria based on other taxa
Criterion 9: A wetland should be considered internationally important if it regularly
supports 1% of the individuals in a population of one species or subspecies of wetland-
dependent non-avian animal species.

86. Consider the following statements about Sundarbans:


1. It is the largest mangrove delta of the world.
2. It was declared a tiger reserve in 1983.
3. The mangroves in Sundarbans serve as nurseries to shell fish and fin-fishes.
Which of the statements given above is/are correct?
(a) 1 only
(b) 1 and 3 only
(c) 2 and 3 only
(d) 1, 2 and 3

Solution: (b)
Sundarban is the largest mangrove delta of the world and encompasses over hundreds
of islands, with a maze of innumerable rivers, rivulets, and creeks. Hence Statement 1
is correct. The name ‘Sundarban’ means “beautiful forest” and it is believed to be
derived from a mangrove tree species ‘Sundari’ (Heritiera fomes). The Indian
Sundarban is the southernmost part of the estuarine delta formed by the River Ganges
and Brahmaputra, bordering the Bay of Bengal.
Sundarban Tiger Reserve is one of the initial nine tiger reserves declared during 1973
and encompasses a total area of 2584.89 km2 of which 1699.62 km2 has been declared
as the Critical Tiger Habitat and 885.27 km2 as the buffer area. Hence Statement 2 is
incorrect. The Sundarbans constitutes over 60% of the total mangrove forest area in
the entire country and has 90% of the total Indian mangrove species. These comprise of
true mangroves or major elements, minor elements of mangroves or and mangrove
associates, black mangrove trees and shrubs, non-halophytic non-mangrove associates
in the area, halophytic herbs, shrubs, and weeds and epiphytic and parasitic plants. The
mangrove forests act as a natural shelter belt and protect the hinterland from storms,
cyclones, tidal surges, sea water seepage and intrusion. The mangroves serve as
nurseries to shell fish and fin-fishes and sustain the coastal fisheries of the entire
eastern coast. Hence Statement 3 is correct.

87. Consider the following:


1. East Kolkata Wetlands
2. Tso Morari
3. Chandratal
Which of those given above is/are Wetlands of International importance?
(a) 1 only
(b) 2 and 3 only
(c) 1 and 3 only
(d) 1, 2 and 3

Solution: (d)
List of wetlands of international Importance in India
India has designated Sundarban Wetland as a Wetland of International Importance, its
27th. The Site (Ramsar Site no. 2370) is located within the largest mangrove forest in
the world, the Sundarbans, that encompasses hundreds of islands and a maze of rivers,
rivulets and creeks, in the delta of the Rivers Ganges and Brahmaputra on the Bay of
Bengal in India and Bangladesh.

88. Consider the following statements:


1. Desertification causes a relatively dry land region to become increasingly arid.
2. Cold desert areas do not undergo desertification.
Which of the statements given above is/are correct?
(a) 1 only
(b) 2 only
(c) Both 1 and 2
(d) Neither 1 nor 2
Solution: (a)
Desertification is defined as “a type of land degradation in which a relatively dry land
region becomes increasingly arid, typically losing its bodies of water as well as
vegetation and wildlife.” Hence Statement 1 is correct.
Loss of soil cover, mainly due to rainfall and surface runoff, is one of the biggest reasons
for desertification. It is responsible for 10.98 per cent of desertification in the country.
Water erosion is observed in both hot and cold desert areas, across various land covers
and with varying levels of severity. The next big reason is wind erosion. Hence
Statement 2 is incorrect.

89. Consider the following :


1. Bottom trawling fishing boats.
2. Decreasing level of nitrogen in seawater.
3. Destruction of Mangroves.
Which of these given above is/are threats to coral habitats?
(a) 1 only
(b) 1 and 3
(c) 3 only
(d) 2 and 3

Solution: (b)
Major threats to coral reefs and their habitats include:
1. Climate change: Corals cannot survive if the water temperature is too high. Global
warming has already led to increased levels of coral bleaching, and this is predicted to
increase in frequency and severity in the coming decades.
2. Destructive fishing practices: These include cyanide fishing, blast or dynamite fishing,
bottom trawling, and muro-ami (banging on the reef with sticks). Bottom-trawling is
one of the greatest threats to cold-water coral reefs. Hence Statement 1 is correct.
3. Overfishing: This affects the ecological balance of coral reef communities, warping
the food chain and causing effects far beyond the directly overfished population.
4. Careless tourism: Careless boating, diving, snorkeling, and fishing happens around
the world, with people touching reefs, stirring up sediment, collecting coral, and
dropping anchors on reefs. Some tourist resorts and infrastructure have been built
directly on top of reefs, and some resorts empty their sewage or other wastes directly
into water surrounding coral reefs.
5. Pollution: Urban and industrial waste, sewage, agrochemicals, and oil pollution are
poisoning reefs. These toxins are dumped directly into the ocean or carried by river
systems from sources upstream. Some pollutants, such as sewage and runoff from
farming, increase the level of nitrogen in seawater, causing an overgrowth of algae,
which 'smothers' reefs by cutting off their sunlight. Hence Statement 2 is incorrect.
6. Sedimentation: Erosion caused by construction (both along coasts and inland),
mining, logging, and farming is leading to increased sediment in rivers. This ends up in
the ocean, where it can 'smother' corals by depriving them of the light needed to
survive. The destruction of mangrove forests, which normally trap large amounts of
sediment, is exacerbating the problem. Hence Statement 3 is correct.
7. Coral mining: Live coral is removed from reefs for use as bricks, road-fill, or cement
for new buildings. Corals are also sold as souvenirs to tourists and to exporters who
don't know or don't care about the longer term damage done, and harvested for the live
rock trade.
90. Consider the following statements about Corals:
1. Coral polyps can capture small organisms that swim too close, to feed themselves.
2. Shallow water corals can photosynthesize due to presence of chlorophyll in their
bodies.
Which of the statements given above is/are correct?
(a) 1 only
(b) 2 only
(c) Both 1 and 2
(d) Neither 1 nor 2

Solution: (a)
Corals are related to sea anemones, and they all share the same simple structure, the
polyp. The polyp is like a tin can open at just one end: the open end has a mouth
surrounded by a ring of tentacles. The tentacles have stinging cells, called nematocysts,
which allow the coral polyp to capture small organisms that swim too close. Hence
Statement 1 is correct. Inside the body of the polyp are digestive and reproductive
tissues. Corals differ from sea anemones in their production of a mineral skeleton.
Shallow water corals that live in warm water often have another source of food, the
zooxanthellae. These single-celled algae photosynthesize and pass some of the food they
make from the sun’s energy to their hosts, and in exchange the coral animal gives
nutrients to the algae. Hence Statement 2 is incorrect. It is this relationship that
allows shallow water corals to grow fast enough to build the enormous structures we
call reefs. The zooxanthellae also provide much of the green, brown, and reddish colors
that corals have.

91. Consider the following statements about The National Green Tribunal:
1. It is a specialized body equipped with the necessary expertise to handle
environmental disputes
2. It is mandated to dispose of applications or appeals finally within six months of filing
of the same
Which of the statements given above is/are correct?
(a) 1 only
(b) 2 only
(c) Both 1 and 2
(d) Neither 1 nor 2

Solution: (c)
The National Green Tribunal has been established on 18.10.2010 under the National
Green Tribunal Act 2010 for effective and expeditious disposal of cases relating to
environmental protection and conservation of forests and other natural resources
including enforcement of any legal right relating to environment and giving relief and
compensation for damages to persons and property and for matters connected
therewith or incidental thereto. It is a specialized body equipped with the necessary
expertise to handle environmental disputes involving multi-disciplinary issues. Hence
Statement 1 is correct. The Tribunal shall not be bound by the procedure laid down
under the Code of Civil Procedure, 1908, but shall be guided by principles of natural
justice.
The Tribunal's dedicated jurisdiction in environmental matters shall provide speedy
environmental justice and help reduce the burden of litigation in the higher courts. The
Tribunal is mandated to make and endeavour for disposal of applications or appeals
finally within 6 months of filing of the same. Hence Statement 2 is correct.

92. Consider the following statements regarding Coastal Regulation Zone (CRZ)
Notification, 2018:
1. It will further add to creating additional opportunities for affordable housing.
2. For CRZ-II (Urban) areas, Floor Space Index (FSI) or the Floor Area Ratio (FAR) had
been frozen.
3. For CRZ-III (Rural) areas, two separate categories have been created on the basis of
population density.
4. Temporary tourism facilities are strictly prohibited in the "No Development Zone"
(NDZ) of the CRZ-III areas
Which of the statements given above is/are correct?
(a) 1 and 3
(b) 2 only
(c) 2 and 4
(d) 1, 3 and 4

Solution: (a)
With the objective of conservation and protection of the coastal environment, Ministry
of Environment and Forest and Climate Change notified the Coastal Regulation Zone
Notification in 1991, which was subsequently revised in 2011. The notification was
amended from time to time based on representations received. The Ministry of
Environment, Forest & Climate Change constituted a Committee in June 2014 under the
Chairmanship of Dr. Shailesh Nayak to examine the various issues and concerns of
Coastal States/UTs and other stakeholders for recommending appropriate changes in
the CRZ Notification, 2011.
A draft notification was issued in April, 2018 for inviting comments from public at
large. CRZ Notification will further add to creating additional opportunities for
affordable housing. Hence Statement 1 is correct. This will benefit not only the
housing sector but the people at large looking for shelter. The new Notification will
boost tourism in terms of more activities, more infrastructure and more opportunities
and will certainly go a long way in creating employment opportunities in various
aspects of tourism.
Salient Features:
(i) Allowing FSI as per current norms in CRZ areas: As per CRZ, 2011 Notification, for
CRZ-II (Urban) areas, Floor Space Index (FSI) or the Floor Area Ratio (FAR) had been
frozen as per 1991 Development Control Regulation (DCR) levels.
In the CRZ, 2018 Notification, it has been decided to de-freeze the same and permit FSI
for construction projects, as prevailing on the date of the new Notification. This will
enable redevelopment of these areas to meet the emerging needs. Hence Statement 2
is incorrect.
(ii) Densely populated rural areas to be afforded greater opportunity for
development: For CRZ-III (Rural) areas, two separate categories have now been
stipulated as below:
(a) CRZ-III A - These are densely populated rural areas with a population density of
2161 per square kilometre as per 2011 Census. Such areas shall have a No Development
Zone (NDZ) of 50 meters from the HTL as against 200 meters from the High Tide Line
stipulated in the CRZ Notification, 2011 since such areas have similar characteristics as
urban areas.
(b) CRZ-III B - Rural areas with population density of below 2161 per square kilometre
as per 2011 Census. Such areas shall continue to have an NDZ of 200 meters from the
HTL. Hence Statement 3 is correct.
(iii) Tourism infrastructure for basic amenities to be promoted: Temporary
tourism facilities such as shacks, toilet blocks, change rooms, drinking water facilities
etc. have now been permitted in Beaches. Such temporary tourism facilities are also
now permissible in the "No Development Zone" (NDZ) of the CRZ-III areas as per the
Notification. However, a minimum distance of 10 m from HTL should be maintained for
setting up of such facilities. Hence Statement 4 is incorrect.
(iv) CRZ Clearances streamlined: The procedure for CRZ clearances has been
streamlined. Only such projects/activities, which are located in the CRZ-I (Ecologically
Sensitive Areas) and CRZ IV (area covered between Low Tide Line and 12 Nautical Miles
seaward) shall be dealt with for CRZ clearance by the Ministry of Environment, Forest
and Climate Change. The powers for clearances with respect to CRZ-II and III have been
delegated at the State level with necessary guidance.
(v) A No Development Zone (NDZ) of 20 meters has been stipulated for all Islands:
For islands close to the main land coast and for all Backwater Islands in the main land,
in wake of space limitations and unique geography of such regions, bringing uniformity
in treatment of such regions, NDZ of 20 m has been stipulated.
(vi) All Ecologically Sensitive Areas have been accorded special importance:
Specific guidelines related to their conservation and management plans have been
drawn up as a part of the CRZ Notification.
93. Consider the following statements about Forest policy:
1. National Forest Policy, 1988 called for the need to associate tribal people in the
protection, regeneration and development of forests
2. Scheduled Tribes and Other Traditional Forest Dwellers (Recognition of Forest
Rights) Act, 2006, balances the right to environment with the right to life and livelihood
of forest dwellers.
Which of the statements given above is/are correct?
(a) 1 only
(b) 2 only
(c) Both 1 and 2
(d) Neither 1 nor 2

Solution: (c)
The symbiotic relationship between forests and forest-dwelling communities found
recognition in the National Forest Policy, 1988. The policy called for the need to
associate tribal people in the protection, regeneration and development of forests.
Hence Statement 1 is correct. The Scheduled Tribes and Other Traditional Forest
Dwellers (Recognition of Forest Rights) Act, 2006, was enacted to protect the
marginalised socio-economic class of citizens and balance the right to environment with
their right to life and livelihood. Hence Statement 2 is correct. It has been enacted to
recognize and vest the forest rights and occupation of forest land in forest dwelling
Scheduled Tribes and other traditional forest dwellers, who have been residing in such
forests for generations, but whose rights could not be recorded.
94. Consider the following statements about Intergovernmental Panel on Climate
Change (IPCC) :
1. It was created in 1988 by the UN Environment Assembly.
2. It is an organization of only governments that are members of the United Nations.
3. The IPCC does not conduct its own research regarding climate change.
Which of the statements given above is/are not correct?
(a) 1 only
(b) 2 and 3
(c) 1 and 2
(d) 3 only

Solution: (c)
Created in 1988 by the World Meteorological Organization (WMO) and the United
Nations Environment Programme (UNEP), the objective of the Intergovernmental Panel
on Climate Change (IPCC) is to provide governments at all levels with scientific
information that they can use to develop climate policies. Hence Statement 1 is
incorrect. IPCC reports are also a key input into international climate change
negotiations.
The IPCC is an organization of governments that are members of the United Nations or
WMO. Hence Statement 2 is incorrect. The IPCC currently has 195 members.
Thousands of people from all over the world contribute to the work of the IPCC. For the
assessment reports, IPCC scientists volunteer their time to assess the thousands of
scientific papers published each year to provide a comprehensive summary of what is
known about the drivers of climate change, its impacts and future risks, and how
adaptation and mitigation can reduce those risks.
An open and transparent review by experts and governments around the world is an
essential part of the IPCC process, to ensure an objective and complete assessment and
to reflect a diverse range of views and expertise. Through its assessments, the IPCC
identifies the strength of scientific agreement in different areas and indicates where
further research is needed. The IPCC does not conduct its own research. Hence
Statement 3 is correct.

95. Consider the following statements about the Global Environment Facility (GEF):
1. The Global Environment Facility (GEF) was established after the Rio Earth Summit to
address global environmental issues.
2. It is a Financial Mechanism for five major international environmental conventions.
Which of the statements given above is/are correct?
(a) 1 only
(b) 2 only
(c) Both 1 and 2
(d) Neither 1 nor 2

Solution: (b)
The Global Environment Facility (GEF) was established in 1991 and unites 182 member
governments — in partnership with international institutions, nongovernmental
organizations, and the private sector — to address global environmental issues. Hence
Statement 1 is incorrect.
It is a Financial Mechanism for five major international environmental conventions: the
Minamata Convention on Mercury, the Stockholm Convention on Persistent Organic
Pollutants (POPs), the United Nations Convention on Biological Diversity (UNCBD), the
United Nations Convention to Combat Desertification (UNCCD) and the United Nations
Framework Convention on Climate Change (UNFCCC). Hence Statement 2 is correct.

96. What is meant by the term carbon credits?


(a) Tradeable certificate representing right to produce renewable energy
(b) Tradeable certificate representing right to mine coal in a region
(c) Certificate representing right to set up thermal power plant
(d) Tradeable certificate representing right to emit one tonne of carbon or carbon
dioxide equivalent

Solution: (d)
Carbon credit is a tradeable certificate representing right to emit one tonne of carbon or
carbon dioxide equivalent

97. Consider the following statements about the Biological Diversity Act in 2002:
1. The Act is implemented through a three-tiered institutional structure at the national,
state and local levels.
2. The State Biodiversity Boards (SBBs) constituted by the State Governments deal with
all matters relating to access by Indians for commercial purposes.
Which of the statements given above is/are correct?
(a) 1 only
(b) 2 only
(c) Both 1 and 2
(d) Neither 1 nor 2

Solution: (c)
Pursuant to the CBD, India enacted the Biological Diversity Act in 2002, and notified
Biological Diversity Rules in 2004, to give effect to the provisions of this Convention.
The Act is implemented through a three-tiered institutional structure at the national,
state and local levels. Hence Statement 1 is correct. The National Biodiversity
Authority (NBA) has been set up in October, 2003 in Chennai.
The vision of NBA is the conservation and sustainable use of India’s rich biodiversity
and associated knowledge with peoples participation, ensuring the process of benefit
sharing for well being of present and future generations. The mission of NBA is to
ensure effective implementation of Biological Diversity Act, 2002 and the Biological
Diversity Rules 2004 for conservation of biodiversity, sustainable use of its components
and fair and equitable sharing of benefits arising out of utilization of genetic resources.
The NBA interalia deals with all matters relating to requests for access by foreign
individuals, institutions or companies, and transfer of results of research to any
foreigner. The State Biodiversity Boards (SBBs) constituted by the State Governments
deal with all matters relating to access by Indians for commercial purposes. Hence
Statement 2 is correct. The institutions of self-governments are required to set up
Biodiversity Management Committees (BMCs) in their respective areas for
conservation, sustainable use, documentation of biodiversity and chronicling of
knowledge related to biodiversity.
98. Consider the following statements:
1. Appropriate valuation of forests and their services
2. A Credible certification process to enhance value of forest products
3. Development of a national forest ecosystems management information system.
Which of these are strategies included in the Draft forest policy 2018 for forest & tree
cover Management?
(a) 1 only
(b) 1 and 2
(c) 3 only
(d) 1, 2 and 3

Solution: (d)
The draft forest policy, 2018 will be an overarching policy for forest management. It
aims at bringing a minimum of one-third of India’s total geographical area under forest
or tree cover. It seems to address the concern of decline in forest productivity.
The draft has concepts such as:
i. economic valuation of forests
Forests provide a wide range of tangible benefits like timber, fuel wood, fodder and a
wide range of NTFPs and intangible benefits like ecological services, hydrological
benefits, soil conservation, flood control, carbon sequestration, biodiversity
conservation, amelioration of the overall environment, etc. Contribution of the forest to
the national economy is estimated generally on the basis of the recorded removals of
industrial wood, fuel wood and NTFPs, which is abysmally low as compared to the
actuals. Scientific methods will be evolved for appropriate valuation of forests and their
services through institutions of repute.
Hence Statement 1 is correct.
ii. forest certification
A Credible certification process can provide premium on the products, which can
enhance value of forest product harvested sustainably. Adoption of appropriate
certification regimes will be encouraged though phase wise adoption of compatible
standards and institutional framework in forest management.
Hence Statement 2 is correct.
iii. national forest ecosystem management information system
Lack of adequate pan country reliable and compatible datasets in the forestry sector is a
matter of serious concern. Such comprehensive and reliable datasets are essential for
scientific planning and management. Systems need to be designed and put in place to
ensure a regular flow of reliable data from the states and other sources and making it
available in the public domain. A national forest ecosystems management information
system will be developed and made operational using the latest information and
communication technology. Assessment of growing stock & carbon stock in Indian
forests will be given more stress and importance.
Hence Statement 3 is correct.

99. Consider the following statements:


1. Wild Life (Protection) Act 1972 was enacted with the objective of effectively
protecting the wild life of the country and to control poaching, smuggling.
2. State of Jammu and Kashmir is not covered by any Act for Wildlife protection
Which of the statements given above is/are correct?
(a) 1 only
(b) 2 only
(c) Both 1 and 2
(d) Neither 1 nor 2

Solution: (a)
The Government of India enacted Wild Life (Protection) Act 1972 with the objective of
effectively protecting the wild life of this country and to control poaching, smuggling
and illegal trade in wildlife and its derivatives. The Act was amended in January 2003
and punishment and penalty for offences under the Act have been made more stringent.
Hence Statement 1 is correct.
State of Jammu and Kashmir is not covered by the Wildlife protection Act, 1972. It has a
similar law enacted for the purpose. Hence Statement 2 is incorrect.

100. Consider the following statements:


1. Sundarban Wetland is both a Ramsar site and is inscribed on the World Heritage List
under the UNESCO World Heritage Convention.
2. Keoladeo National Park is both a Ramsar site and a UNESCO Biosphere Reserve.
Which of the statements given above is/are correct?
(a) 1 only
(b) 2 only
(c) Both 1 and 2
(d) Neither 1 nor 2

Solution: (a)
Wetlands of International importance in India which are also inscribed on the World
Heritage List under the UNESCO World Heritage Convention:
1. Sundarban Wetland. Hence Statement 1 is correct.
2. Keoladeo National Park
Wetlands of International importance in India which are also UNESCO Biosphere
Reserves:
1. Sundarban Wetland.
Hence Statement 2 is incorrect.

You might also like